You are on page 1of 112

火箭学院 火速提分

新页码1
火箭学院 火速提分
新页码2
火箭学院 火速提分
新页码3
火箭学院 火速提分
新页码4
火箭学院 火速提分
新页码5
火箭学院 火速提分
新页码6
火箭学院 火速提分
新页码7
火箭学院 火速提分
新页码8
火箭学院 火速提分
新页码9
火箭学院 火速提分
新页码10
火箭学院 火速提分
新页码11
火箭学院 火速提分
新页码12
火箭学院 火速提分
新页码13
火箭学院 火速提分
新页码14
火箭学院 火速提分
新页码15
火箭学院 火速提分
新页码16
火箭学院 火速提分
新页码17
火箭学院 火速提分
新页码18
火箭学院 火速提分
新页码19
火箭学院 火速提分
新页码20
火箭学院 火速提分
新页码21
火箭学院 火速提分
新页码22
火箭学院 火速提分
新页码23
火箭学院 火速提分
新页码24
火箭学院 火速提分
新页码25
火箭学院 火速提分
新页码26
火箭学院 火速提分
新页码27
火箭学院 火速提分
新页码28
火箭学院 火速提分
新页码29
火箭学院 火速提分
新页码30
火箭学院 火速提分
新页码31
火箭学院 火速提分
新页码32
火箭学院 火速提分
新页码33
火箭学院 火速提分
新页码34
火箭学院 火速提分
新页码35
火箭学院 火速提分
新页码36
火箭学院 火速提分
新页码37
火箭学院 火速提分
新页码38

AP® Statistics
2007 Free-Response Questions

The College Board: Connecting Students to College Success


The College Board is a not-for-profit membership association whose mission is to connect students to college success and
opportunity. Founded in 1900, the association is composed of more than 5,000 schools, colleges, universities, and other
educational organizations. Each year, the College Board serves seven million students and their parents, 23,000 high schools, and
3,500 colleges through major programs and services in college admissions, guidance, assessment, financial aid, enrollment, and
teaching and learning. Among its best-known programs are the SAT®, the PSAT/NMSQT®, and the Advanced Placement
Program® (AP®). The College Board is committed to the principles of excellence and equity, and that commitment is embodied
in all of its programs, services, activities, and concerns.

© 2007 The College Board. All rights reserved. College Board, Advanced Placement Program, AP, AP Central, SAT, and the
acorn logo are registered trademarks of the College Board. PSAT/NMSQT is a registered trademark of the College Board and
National Merit Scholarship Corporation.
Permission to use copyrighted College Board materials may be requested online at:
www.collegeboard.com/inquiry/cbpermit.html.

Visit the College Board on the Web: www.collegeboard.com.


AP Central is the official online home for the AP Program: apcentral.collegeboard.com.
火箭学院 火速提分
新页码39
®
2007 AP STATISTICS FREE-RESPONSE QUESTIONS

Formulas begin on page 3.


Questions begin on page 6.
Tables begin on page 12.

-2-
火箭学院 火速提分
新页码40
®
2007 AP STATISTICS FREE-RESPONSE QUESTIONS

Formulas

(I) Descriptive Statistics

∑ xi
x =
n

sx =
1
n−1
∑ xi − x
2
d i

dn1 − 1is21 + dn2 − 1is22


sp =
dn1 − 1i + dn2 − 1i
y = b0 + b1 x

b1 =
d
∑ xi − x y i − y id i
d
∑ xi − x i2
b0 = y − b1 x

r =
1 x − x
∑ i
FG IJ FG yi − y IJ
n−1 sx H K H sy K
sy
b1 = r
sx

d
∑ yi − y i i2
sb = n−2
1
d
∑ xi − x i2

-3-
火箭学院 火速提分
新页码41
®
2007 AP STATISTICS FREE-RESPONSE QUESTIONS

(II) Probability

P( A ∪ B) = P( A) + P( B) − P( A ∩ B)

P( A ∩ B )
P( A B ) =
P( B )

E ( X ) = μ x = ∑ xi pi

d
Var( X ) = s 2x = ∑ xi − μ x i2 pi

If X has a binomial distribution with parameters n and p, then:

P( X = k ) =
FG nIJ p k (1 − p)n − k
H kK
μ x = np

sx = np(1 − p)

μ = p
p

p(1 − p)
s =
p n

If x is the mean of a random sample of size n from an infinite


population with mean μ and standard deviation s, then:

μx = μ

s
sx =
n

-4-
火箭学院 火速提分
新页码42
®
2007 AP STATISTICS FREE-RESPONSE QUESTIONS

(III) Inferential Statistics

statistic - parameter
Standardized test statistic:
standard deviation of statistic

Confidence interval: statistic ± (critical value ) ∑ (standard deviation of statistic)

Single-Sample

Standard Deviation
Statistic
of Statistic
σ
Sample Mean n

p(1 − p)
Sample Proportion n

Two-Sample

Standard Deviation
Statistic
of Statistic

Difference of σ 12 σ 22
sample means +
n1 n2

Special case when σ 1 = σ 2

1 1
σ +
n1 n2

Difference of p1 (1 − p1 ) p (1 − p2 )
sample proportions + 2
n1 n2

Special case when p1 = p2

b g
p 1− p
1
+
1
n1 n2

Chi-square test statistic = ∑


aobserved − expectedf 2

expected

-5-
火箭学院 火速提分
新页码43
®
2007 AP STATISTICS FREE-RESPONSE QUESTIONS

STATISTICS
SECTION II
Part A
Questions 1-5
Spend about 65 minutes on this part of the exam.
Percent of Section II grade—75

Directions: Show all your work. Indicate clearly the methods you use, because you will be graded on the
correctness of your methods as well as on the accuracy and completeness of your results and explanations.

1. The department of agriculture at a university was interested in determining whether a preservative was effective
in reducing discoloration in frozen strawberries. A sample of 50 ripe strawberries was prepared for freezing.
Then the sample was randomly divided into two groups of 25 strawberries each. Each strawberry was placed
into a small plastic bag.
The 25 bags in the control group were sealed. The preservative was added to the 25 bags containing strawberries
in the treatment group, and then those bags were sealed. All bags were stored at 0⬚C for a period of 6 months. At
the end of this time, after the strawberries were thawed, a technician rated each strawberry’s discoloration from
1 to 10, with a low score indicating little discoloration.
The dotplots below show the distributions of discoloration rating for the control and treatment groups.

(a) The standard deviation of ratings for the control group is 2.141. Explain how this value summarizes
variability in the control group.
(b) Based on the dotplots, comment on the effectiveness of the preservative in lowering the amount of
discoloration in strawberries. (No calculations are necessary.)
(c) Researchers at the university decided to calculate a 95 percent confidence interval for the difference in mean
discoloration rating between strawberries that were not treated with preservative and those that were treated
with preservative. The confidence interval they obtained was (0.16, 2.72). Assume that the conditions
necessary for the t-confidence interval are met.
Based on the confidence interval, comment on whether there would be a difference in the population mean
discoloration ratings for the treated and untreated strawberries.

© 2007 The College Board. All rights reserved.


Visit apcentral.collegeboard.com (for AP professionals) and www.collegeboard.com/apstudents (for students and parents).

GO ON TO THE NEXT PAGE.


-6-
火箭学院 火速提分
新页码44
®
2007 AP STATISTICS FREE-RESPONSE QUESTIONS

2. As dogs age, diminished joint and hip health may lead to joint pain and thus reduce a dog’s activity level. Such
a reduction in activity can lead to other health concerns such as weight gain and lethargy due to lack of exercise.
A study is to be conducted to see which of two dietary supplements, glucosamine or chondroitin, is more
effective in promoting joint and hip health and reducing the onset of canine osteoarthritis. Researchers will
randomly select a total of 300 dogs from ten different large veterinary practices around the country. All of the
dogs are more than 6 years old, and their owners have given consent to participate in the study. Changes in joint
and hip health will be evaluated after 6 months of treatment.
(a) What would be an advantage to adding a control group in the design of this study?
(b) Assuming a control group is added to the other two groups in the study, explain how you would assign the
300 dogs to these three groups for a completely randomized design.
(c) Rather than using a completely randomized design, one group of researchers proposes blocking on clinics,
and another group of researchers proposes blocking on breed of dog. How would you decide which one of
these two variables to use as a blocking variable?

3. Big Town Fisheries recently stocked a new lake in a city park with 2,000 fish of various sizes. The distribution
of the lengths of these fish is approximately normal.
(a) Big Town Fisheries claims that the mean length of the fish is 8 inches. If the claim is true, which of the
following would be more likely?
• A random sample of 15 fish having a mean length that is greater than 10 inches
or
• A random sample of 50 fish having a mean length that is greater than 10 inches
Justify your answer.
(b) Suppose the standard deviation of the sampling distribution of the sample mean for random samples of size
50 is 0.3 inch. If the mean length of the fish is 8 inches, use the normal distribution to compute the
probability that a random sample of 50 fish will have a mean length less than 7.5 inches.
(c) Suppose the distribution of fish lengths in this lake was nonnormal but had the same mean and standard
deviation. Would it still be appropriate to use the normal distribution to compute the probability in part (b) ?
Justify your answer.

© 2007 The College Board. All rights reserved.


Visit apcentral.collegeboard.com (for AP professionals) and www.collegeboard.com/apstudents (for students and parents).

GO ON TO THE NEXT PAGE.


-7-
火箭学院 火速提分
新页码45
®
2007 AP STATISTICS FREE-RESPONSE QUESTIONS

4. Investigators at the U.S. Department of Agriculture wished to compare methods of determining the level
of E. coli bacteria contamination in beef. Two different methods (A and B) of determining the level of
contamination were used on each of ten randomly selected specimens of a certain type of beef. The data
obtained, in millimicrobes/liter of ground beef, for each of the methods are shown in the table below.

Specimen
1 2 3 4 5 6 7 8 9 10
A 22.7 23.6 24.0 27.1 27.4 27.8 34.4 35.2 40.4 46.8
Method
B 23.0 23.1 23.7 26.5 26.6 27.1 33.2 35.0 40.5 47.8

Is there a significant difference in the mean amount of E. coli bacteria detected by the two methods for this
type of beef? Provide a statistical justification to support your answer.

© 2007 The College Board. All rights reserved.


Visit apcentral.collegeboard.com (for AP professionals) and www.collegeboard.com/apstudents (for students and parents).

GO ON TO THE NEXT PAGE.


-8-
火箭学院 火速提分
新页码46
®
2007 AP STATISTICS FREE-RESPONSE QUESTIONS

5. Researchers want to determine whether drivers are significantly more distracted while driving when using a
cell phone than when talking to a passenger in the car. In a study involving 48 people, 24 people were randomly
assigned to drive in a driving simulator while using a cell phone. The remaining 24 were assigned to drive in
the driving simulator while talking to a passenger in the simulator. Part of the driving simulation for both groups
involved asking drivers to exit the freeway at a particular exit. In the study, 7 of the 24 cell phone users missed
the exit, while 2 of the 24 talking to a passenger missed the exit.
(a) Would this study be classified as an experiment or an observational study? Provide an explanation to support
your answer.
(b) State the null and alternative hypotheses of interest to the researchers.
(c) One test of significance that you might consider using to answer the researchers’ question is a two-sample
z-test. State the conditions required for this test to be appropriate. Then comment on whether each condition
is met.
(d) Using an advanced statistical method for small samples to test the hypotheses in part (b), the researchers
report a p-value of 0.0683. Interpret, in everyday language, what this p-value measures in the context
of this study and state what conclusion should be made based on this p-value.

© 2007 The College Board. All rights reserved.


Visit apcentral.collegeboard.com (for AP professionals) and www.collegeboard.com/apstudents (for students and parents).

GO ON TO THE NEXT PAGE.


-9-
火箭学院 火速提分
新页码47
®
2007 AP STATISTICS FREE-RESPONSE QUESTIONS

STATISTICS
SECTION II
Part B
Question 6
Spend about 25 minutes on this part of the exam.
Percent of Section II grade—25

Directions: Show all your work. Indicate clearly the methods you use, because you will be graded on the
correctness of your methods as well as on the accuracy and completeness of your results and explanations.

6. A study was designed to explore subjects’ ability to judge the distance between two objects placed in a dimly lit
room. The researcher suspected that the subjects would generally overestimate the distance between the objects
in the room and that this overestimation would increase the farther apart the objects were.
The two objects were placed at random locations in the room before a subject estimated the distance (in feet)
between those two objects. After each subject estimated the distance, the locations of the objects were
rerandomized before the next subject viewed the room.
After data were collected for 40 subjects, two linear models were fit in an attempt to describe the relationship
between the subjects’ perceived distances (y) and the actual distance, in feet, between the two objects.
Model 1: yˆ = 0.238 + 1.080 ⫻ (actual distance)
The standard errors of the estimated coefficients for Model 1 are 0.260 and 0.118, respectively.
Model 2: yˆ = 1.102 ⫻ (actual distance)
The standard error of the estimated coefficient for Model 2 is 0.393.
(a) Provide an interpretation in context for the estimated slope in Model 1.
(b) Explain why the researcher might prefer Model 2 to Model 1 in this context.
(c) Using Model 2, test the researcher’s hypothesis that in dim light participants overestimate the distance, with
the overestimate increasing as the actual distance increases. (Assume appropriate conditions for inference
are met.)

The researchers also wanted to explore whether the performance on this task differed between subjects who wear
contact lenses and subjects who do not wear contact lenses. A new variable was created to indicate whether or
not a subject wears contact lenses. The data for this variable were coded numerically (1 = contact wearer,
0 = noncontact wearer), and this new variable, named “contact,” was included in the following model.
Model 3: yˆ = 1.05 ⫻ (actual distance) + 0.12 ⫻ (contact) ⫻ (actual distance )
The standard errors of the estimated coefficients for Model 3 are 0.357 and 0.032, respectively.

© 2007 The College Board. All rights reserved.


Visit apcentral.collegeboard.com (for AP professionals) and www.collegeboard.com/apstudents (for students and parents).

GO ON TO THE NEXT PAGE.


-10-
火箭学院 火速提分
新页码48
®
2007 AP STATISTICS FREE-RESPONSE QUESTIONS

(d) Using Model 3, sketch the estimated regression model for contact wearers and the estimated regression
model for noncontact wearers on the grid below.

(e) In the context of this study, provide an interpretation of the estimated coefficients for Model 3.

STOP

END OF EXAM

© 2007 The College Board. All rights reserved.


Visit apcentral.collegeboard.com (for AP professionals) and www.collegeboard.com/apstudents (for students and parents).

-11-
火箭学院 火速提分
新页码49
®
2007 AP STATISTICS FREE-RESPONSE QUESTIONS

Probability

Table entry for z is the


probability lying below z.
z

Table A Standard normal probabilities

z .00 .01 .02 .03 .04 .05 .06 .07 .08 .09
– 3.4 .0003 .0003 .0003 .0003 .0003 .0003 .0003 .0003 .0003 .0002
– 3.3 .0005 .0005 .0005 .0004 .0004 .0004 .0004 .0004 .0004 .0003
– 3.2 .0007 .0007 .0006 .0006 .0006 .0006 .0006 .0005 .0005 .0005
– 3.1 .0010 .0009 .0009 .0009 .0008 .0008 .0008 .0008 .0007 .0007
– 3.0 .0013 .0013 .0013 .0012 .0012 .0011 .0011 .0011 .0010 .0010
– 2.9 .0019 .0018 .0018 .0017 .0016 .0016 .0015 .0015 .0014 .0014
– 2.8 .0026 .0025 .0024 .0023 .0023 .0022 .0021 .0021 .0020 .0019
– 2.7 .0035 .0034 .0033 .0032 .0031 .0030 .0029 .0028 .0027 .0026
– 2.6 .0047 .0045 .0044 .0043 .0041 .0040 .0039 .0038 .0037 .0036
– 2.5 .0062 .0060 .0059 .0057 .0055 .0054 .0052 .0051 .0049 .0048
– 2.4 .0082 .0080 .0078 .0075 .0073 .0071 .0069 .0068 .0066 .0064
– 2.3 .0107 .0104 .0102 .0099 .0096 .0094 .0091 .0089 .0087 .0084
– 2.2 .0139 .0136 .0132 .0129 .0125 .0122 .0119 .0116 .0113 .0110
– 2.1 .0179 .0174 .0170 .0166 .0162 .0158 .0154 .0150 .0146 .0143
– 2.0 .0228 .0222 .0217 .0212 .0207 .0202 .0197 .0192 .0188 .0183
– 1.9 .0287 .0281 .0274 .0268 .0262 .0256 .0250 .0244 .0239 .0233
– 1.8 .0359 .0351 .0344 .0336 .0329 .0322 .0314 .0307 .0301 .0294
– 1.7 .0446 .0436 .0427 .0418 .0409 .0401 .0392 .0384 .0375 .0367
– 1.6 .0548 .0537 .0526 .0516 .0505 .0495 .0485 .0475 .0465 .0455
– 1.5 .0668 .0655 .0643 .0630 .0618 .0606 .0594 .0582 .0571 .0559
– 1.4 .0808 .0793 .0778 .0764 .0749 .0735 .0721 .0708 .0694 .0681
– 1.3 .0968 .0951 .0934 .0918 .0901 .0885 .0869 .0853 .0838 .0823
– 1.2 .1151 .1131 .1112 .1093 .1075 .1056 .1038 .1020 .1003 .0985
– 1.1 .1357 .1335 .1314 .1292 .1271 .1251 .1230 .1210 .1190 .1170
– 1.0 .1587 .1562 .1539 .1515 .1492 .1469 .1446 .1423 .1401 .1379
– 0.9 .1841 .1814 .1788 .1762 .1736 .1711 .1685 .1660 .1635 .1611
– 0.8 .2119 .2090 .2061 .2033 .2005 .1977 .1949 .1922 .1894 .1867
– 0.7 .2420 .2389 .2358 .2327 .2296 .2266 .2236 .2206 .2177 .2148
– 0.6 .2743 .2709 .2676 .2643 .2611 .2578 .2546 .2514 .2483 .2451
– 0.5 .3085 .3050 .3015 .2981 .2946 .2912 .2877 .2843 .2810 .2776
– 0.4 .3446 .3409 .3372 .3336 .3300 .3264 .3228 .3192 .3156 .3121
– 0.3 .3821 .3783 .3745 .3707 .3669 .3632 .3594 .3557 .3520 .3483
– 0.2 .4207 .4168 .4129 .4090 .4052 .4013 .3974 .3936 .3897 .3859
– 0.1 .4602 .4562 .4522 .4483 .4443 .4404 .4364 .4325 .4286 .4247
– 0.0 .5000 .4960 .4920 .4880 .4840 .4801 .4761 .4721 .4681 .4641

-12-
火箭学院 火速提分
新页码50
®
2007 AP STATISTICS FREE-RESPONSE QUESTIONS

Probability

Table entry for z is the


probability lying below z.
z
Table A (Continued)

z .00 .01 .02 .03 .04 .05 .06 .07 .08 .09
0.0 .5000 .5040 .5080 .5120 .5160 .5199 .5239 .5279 .5319 .5359
0.1 .5398 .5438 .5478 .5517 .5557 .5596 .5636 .5675 .5714 .5753
0.2 .5793 .5832 .5871 .5910 .5948 .5987 .6026 .6064 .6103 .6141
0.3 .6179 .6217 .6255 .6293 .6331 .6368 .6406 .6443 .6480 .6517
0.4 .6554 .6591 .6628 .6664 .6700 .6736 .6772 .6808 .6844 .6879
0.5 .6915 .6950 .6985 .7019 .7054 .7088 .7123 .7157 .7190 .7224
0.6 .7257 .7291 .7324 .7357 .7389 .7422 .7454 .7486 .7517 .7549
0.7 .7580 .7611 .7642 .7673 .7704 .7734 .7764 .7794 .7823 .7852
0.8 .7881 .7910 .7939 .7967 .7995 .8023 .8051 .8078 .8106 .8133
0.9 .8159 .8186 .8212 .8238 .8264 .8289 .8315 .8340 .8365 .8389
1.0 .8413 .8438 .8461 .8485 .8508 .8531 .8554 .8577 .8599 .8621
1.1 .8643 .8665 .8686 .8708 .8729 .8749 .8770 .8790 .8810 .8830
1.2 .8849 .8869 .8888 .8907 .8925 .8944 .8962 .8980 .8997 .9015
1.3 .9032 .9049 .9066 .9082 .9099 .9115 .9131 .9147 .9162 .9177
1.4 .9192 .9207 .9222 .9236 .9251 .9265 .9279 .9292 .9306 .9319
1.5 .9332 .9345 .9357 .9370 .9382 .9394 .9406 .9418 .9429 .9441
1.6 .9452 .9463 .9474 .9484 .9495 .9505 .9515 .9525 .9535 .9545
1.7 .9554 .9564 .9573 .9582 .9591 .9599 .9608 .9616 .9625 .9633
1.8 .9641 .9649 .9656 .9664 .9671 .9678 .9686 .9693 .9699 .9706
1.9 .9713 .9719 .9726 .9732 .9738 .9744 .9750 .9756 .9761 .9767
2.0 .9772 .9778 .9783 .9788 .9793 .9798 .9803 .9808 .9812 .9817
2.1 .9821 .9826 .9830 .9834 .9838 .9842 .9846 .9850 .9854 .9857
2.2 .9861 .9864 .9868 .9871 .9875 .9878 .9881 .9884 .9887 .9890
2.3 .9893 .9896 .9898 .9901 .9904 .9906 .9909 .9911 .9913 .9916
2.4 .9918 .9920 .9922 .9925 .9927 .9929 .9931 .9932 .9934 .9936
2.5 .9938 .9940 .9941 .9943 .9945 .9946 .9948 .9949 .9951 .9952
2.6 .9953 .9955 .9956 .9957 .9959 .9960 .9961 .9962 .9963 .9964
2.7 .9965 .9966 .9967 .9968 .9969 .9970 .9971 .9972 .9973 .9974
2.8 .9974 .9975 .9976 .9977 .9977 .9978 .9979 .9979 .9980 .9981
2.9 .9981 .9982 .9982 .9983 .9984 .9984 .9985 .9985 .9986 .9986
3.0 .9987 .9987 .9987 .9988 .9988 .9989 .9989 .9989 .9990 .9990
3.1 .9990 .9991 .9991 .9991 .9992 .9992 .9992 .9992 .9993 .9993
3.2 .9993 .9993 .9994 .9994 .9994 .9994 .9994 .9995 .9995 .9995
3.3 .9995 .9995 .9995 .9996 .9996 .9996 .9996 .9996 .9996 .9997
3.4 .9997 .9997 .9997 .9997 .9997 .9997 .9997 .9997 .9997 .9998

-13-
火箭学院 火速提分
新页码51
®
2007 AP STATISTICS FREE-RESPONSE QUESTIONS

Table entry for p and


C is the point t* with Probability p
probability p lying
above it and
probability C lying
between −t * and t*.

t*

Table B t distribution critical values

Tail probability p
df .25 .20 .15 .10 .05 .025 .02 .01 .005 .0025 .001 .0005
1 1.000 1.376 1.963 3.078 6.314 12.71 15.89 31.82 63.66 127.3 318.3 636.6
2 .816 1.061 1.386 1.886 2.920 4.303 4.849 6.965 9.925 14.09 22.33 31.60
3 .765 .978 1.250 1.638 2.353 3.182 3.482 4.541 5.841 7.453 10.21 12.92
4 .741 .941 1.190 1.533 2.132 2.776 2.999 3.747 4.604 5.598 7.173 8.610
5 .727 .920 1.156 1.476 2.015 2.571 2.757 3.365 4.032 4.773 5.893 6.869
6 .718 .906 1.134 1.440 1.943 2.447 2.612 3.143 3.707 4.317 5.208 5.959
7 .711 .896 1.119 1.415 1.895 2.365 2.517 2.998 3.499 4.029 4.785 5.408
8 .706 .889 1.108 1.397 1.860 2.306 2.449 2.896 3.355 3.833 4.501 5.041
9 .703 .883 1.100 1.383 1.833 2.262 2.398 2.821 3.250 3.690 4.297 4.781
10 .700 .879 1.093 1.372 1.812 2.228 2.359 2.764 3.169 3.581 4.144 4.587
11 .697 .876 1.088 1.363 1.796 2.201 2.328 2.718 3.106 3.497 4.025 4.437
12 .695 .873 1.083 1.356 1.782 2.179 2.303 2.681 3.055 3.428 3.930 4.318
13 .694 .870 1.079 1.350 1.771 2.160 2.282 2.650 3.012 3.372 3.852 4.221
14 .692 .868 1.076 1.345 1.761 2.145 2.264 2.624 2.977 3.326 3.787 4.140
15 .691 .866 1.074 1.341 1.753 2.131 2.249 2.602 2.947 3.286 3.733 4.073
16 .690 .865 1.071 1.337 1.746 2.120 2.235 2.583 2.921 3.252 3.686 4.015
17 .689 .863 1.069 1.333 1.740 2.110 2.224 2.567 2.898 3.222 3.646 3.965
18 .688 .862 1.067 1.330 1.734 2.101 2.214 2.552 2.878 3.197 3.611 3.922
19 .688 .861 1.066 1.328 1.729 2.093 2.205 2.539 2.861 3.174 3.579 3.883
20 .687 .860 1.064 1.325 1.725 2.086 2.197 2.528 2.845 3.153 3.552 3.850
21 .686 .859 1.063 1.323 1.721 2.080 2.189 2.518 2.831 3.135 3.527 3.819
22 .686 .858 1.061 1.321 1.717 2.074 2.183 2.508 2.819 3.119 3.505 3.792
23 .685 .858 1.060 1.319 1.714 2.069 2.177 2.500 2.807 3.104 3.485 3.768
24 .685 .857 1.059 1.318 1.711 2.064 2.172 2.492 2.797 3.091 3.467 3.745
25 .684 .856 1.058 1.316 1.708 2.060 2.167 2.485 2.787 3.078 3.450 3.725
26 .684 .856 1.058 1.315 1.706 2.056 2.162 2.479 2.779 3.067 3.435 3.707
27 .684 .855 1.057 1.314 1.703 2.052 2.158 2.473 2.771 3.057 3.421 3.690
28 .683 .855 1.056 1.313 1.701 2.048 2.154 2.467 2.763 3.047 3.408 3.674
29 .683 .854 1.055 1.311 1.699 2.045 2.150 2.462 2.756 3.038 3.396 3.659
30 .683 .854 1.055 1.310 1.697 2.042 2.147 2.457 2.750 3.030 3.385 3.646
40 .681 .851 1.050 1.303 1.684 2.021 2.123 2.423 2.704 2.971 3.307 3.551
50 .679 .849 1.047 1.299 1.676 2.009 2.109 2.403 2.678 2.937 3.261 3.496
60 .679 .848 1.045 1.296 1.671 2.000 2.099 2.390 2.660 2.915 3.232 3.460
80 .678 .846 1.043 1.292 1.664 1.990 2.088 2.374 2.639 2.887 3.195 3.416
100 .677 .845 1.042 1.290 1.660 1.984 2.081 2.364 2.626 2.871 3.174 3.390
1000 .675 .842 1.037 1.282 1.646 1.962 2.056 2.330 2.581 2.813 3.098 3.300
⬁ .674 .841 1.036 1.282 1.645 1.960 2.054 2.326 2.576 2.807 3.091 3.291

50% 60% 70% 80% 90% 95% 96% 98% 99% 99.5% 99.8% 99.9%
Confidence level C

-14-
火箭学院 火速提分
新页码52
®
2007 AP STATISTICS FREE-RESPONSE QUESTIONS

Probability p
Table entry for p is the point
( χ 2 ) with probability p lying
above it.

(χ2 )

Table C χ 2 critical values


Tail probability p
df .25 .20 .15 .10 .05 .025 .02 .01 .005 .0025 .001 .0005
1 1.32 1.64 2.07 2.71 3.84 5.02 5.41 6.63 7.88 9.14 10.83 12.12
2 2.77 3.22 3.79 4.61 5.99 7.38 7.82 9.21 10.60 11.98 13.82 15.20
3 4.11 4.64 5.32 6.25 7.81 9.35 9.84 11.34 12.84 14.32 16.27 17.73
4 5.39 5.99 6.74 7.78 9.49 11.14 11.67 13.28 14.86 16.42 18.47 20.00
5 6.63 7.29 8.12 9.24 11.07 12.83 13.39 15.09 16.75 18.39 20.51 22.11
6 7.84 8.56 9.45 10.64 12.59 14.45 15.03 16.81 18.55 20.25 22.46 24.10
7 9.04 9.80 10.75 12.02 14.07 16.01 16.62 18.48 20.28 22.04 24.32 26.02
8 10.22 11.03 12.03 13.36 15.51 17.53 18.17 20.09 21.95 23.77 26.12 27.87
9 11.39 12.24 13.29 14.68 16.92 19.02 19.68 21.67 23.59 25.46 27.88 29.67
10 12.55 13.44 14.53 15.99 18.31 20.48 21.16 23.21 25.19 27.11 29.59 31.42
11 13.70 14.63 15.77 17.28 19.68 21.92 22.62 24.72 26.76 28.73 31.26 33.14
12 14.85 15.81 16.99 18.55 21.03 23.34 24.05 26.22 28.30 30.32 32.91 34.82
13 15.98 16.98 18.20 19.81 22.36 24.74 25.47 27.69 29.82 31.88 34.53 36.48
14 17.12 18.15 19.41 21.06 23.68 26.12 26.87 29.14 31.32 33.43 36.12 38.11
15 18.25 19.31 20.60 22.31 25.00 27.49 28.26 30.58 32.80 34.95 37.70 39.72
16 19.37 20.47 21.79 23.54 26.30 28.85 29.63 32.00 34.27 36.46 39.25 41.31
17 20.49 21.61 22.98 24.77 27.59 30.19 31.00 33.41 35.72 37.95 40.79 42.88
18 21.60 22.76 24.16 25.99 28.87 31.53 32.35 34.81 37.16 39.42 42.31 44.43
19 22.72 23.90 25.33 27.20 30.14 32.85 33.69 36.19 38.58 40.88 43.82 45.97
20 23.83 25.04 26.50 28.41 31.41 34.17 35.02 37.57 40.00 42.34 45.31 47.50
21 24.93 26.17 27.66 29.62 32.67 35.48 36.34 38.93 41.40 43.78 46.80 49.01
22 26.04 27.30 28.82 30.81 33.92 36.78 37.66 40.29 42.80 45.20 48.27 50.51
23 27.14 28.43 29.98 32.01 35.17 38.08 38.97 41.64 44.18 46.62 49.73 52.00
24 28.24 29.55 31.13 33.20 36.42 39.36 40.27 42.98 45.56 48.03 51.18 53.48
25 29.34 30.68 32.28 34.38 37.65 40.65 41.57 44.31 46.93 49.44 52.62 54.95
26 30.43 31.79 33.43 35.56 38.89 41.92 42.86 45.64 48.29 50.83 54.05 56.41
27 31.53 32.91 34.57 36.74 40.11 43.19 44.14 46.96 49.64 52.22 55.48 57.86
28 32.62 34.03 35.71 37.92 41.34 44.46 45.42 48.28 50.99 53.59 56.89 59.30
29 33.71 35.14 36.85 39.09 42.56 45.72 46.69 49.59 52.34 54.97 58.30 60.73
30 34.80 36.25 37.99 40.26 43.77 46.98 47.96 50.89 53.67 56.33 59.70 62.16
40 45.62 47.27 49.24 51.81 55.76 59.34 60.44 63.69 66.77 69.70 73.40 76.09
50 56.33 58.16 60.35 63.17 67.50 71.42 72.61 76.15 79.49 82.66 86.66 89.56
60 66.98 68.97 71.34 74.40 79.08 83.30 84.58 88.38 91.95 95.34 99.61 102.7
80 88.13 90.41 93.11 96.58 101.9 106.6 108.1 112.3 116.3 120.1 124.8 128.3
100 109.1 111.7 114.7 118.5 124.3 129.6 131.1 135.8 140.2 144.3 149.4 153.2

-15-
火箭学院 火速提分
新页码53
火箭学院 火速提分
新页码54

AP® Statistics
2007 Scoring Guidelines

The College Board: Connecting Students to College Success


The College Board is a not-for-profit membership association whose mission is to connect students to college success and
opportunity. Founded in 1900, the association is composed of more than 5,000 schools, colleges, universities, and other
educational organizations. Each year, the College Board serves seven million students and their parents, 23,000 high schools, and
3,500 colleges through major programs and services in college admissions, guidance, assessment, financial aid, enrollment, and
teaching and learning. Among its best-known programs are the SAT®, the PSAT/NMSQT®, and the Advanced Placement
Program® (AP®). The College Board is committed to the principles of excellence and equity, and that commitment is embodied
in all of its programs, services, activities, and concerns.

© 2007 The College Board. All rights reserved. College Board, Advanced Placement Program, AP, AP Central, SAT, and the
acorn logo are registered trademarks of the College Board. PSAT/NMSQT is a registered trademark of the College Board and
National Merit Scholarship Corporation.
Permission to use copyrighted College Board materials may be requested online at:
www.collegeboard.com/inquiry/cbpermit.html.

Visit the College Board on the Web: www.collegeboard.com.


AP Central is the official online home for the AP Program: apcentral.collegeboard.com.
火箭学院 火速提分
新页码55

AP® STATISTICS
2007 SCORING GUIDELINES

Question 1

Intent of Question

The goals of this question are to assess a student’s ability to: (1) explain how a commonly used statistic measures
variability; (2) use a graphical display to address the research question of interest in a simple comparative
experiment; and (3) use a confidence interval to make an appropriate inference.

Solution

Part (a):

Roughly speaking, the standard deviation ( s = 2.141) measures a “typical” or “average” distance between
the individual discoloration ratings and the mean discoloration rating for the strawberries in the control group.

Part (b):

The preservative does appear to have been effective in lowering the amount of discoloration in strawberries.
The discoloration ratings for the strawberries that received the preservative, shown in the top dotplot, are
clearly centered at a value that is lower than the center of the discoloration rating distribution for the control
group, shown in the bottom dotplot. In addition, the dotplots can be used to find all five statistics in the five-
number summary (min, Q1, median, Q3, and max) for both groups. In fact, four of the five statistics (the
maximum is the only exception) are lower for the strawberries that received the preservative.

Part (c):

Since zero is not contained in the 95 percent confidence interval for the difference μc − μt , we can conclude
that there is a significant difference between the mean ratings for the two groups at the α = 0.05 level. The
population mean discoloration rating for untreated strawberries is estimated to be between 0.16 and 2.72 units
higher than the population mean discoloration rating for treated strawberries. Thus, we think there would be a
difference in the population mean discoloration ratings for treated and untreated strawberries.

Scoring

Parts (a), (b), and (c) are scored as essentially correct (E), partially correct (P), or incorrect (I).

Part (a) is scored as essentially correct (E) if the standard deviation is interpreted correctly in the context of this
experiment.

Part (a) is scored as partially correct (P) if:


a standard textbook description of the standard deviation is provided without any reference to context, e.g.,
the standard deviation is described as the square root of an average squared deviation, or a “typical” or
“average” deviation from the mean;
OR
the student provides evidence that the distribution of discoloration ratings in the control group is
approximately normal, then correctly applies the 68-95-99.7 rule.

© 2007 The College Board. All rights reserved.


Visit apcentral.collegeboard.com (for AP professionals) and www.collegeboard.com/apstudents (for students and parents).
火箭学院 火速提分
新页码56

AP® STATISTICS
2007 SCORING GUIDELINES

Question 1 (continued)

Part (a) is scored as incorrect (I) if:


the formula for the standard deviation is copied from the formula sheet and no further explanation is
provided;
OR
the student uses the 68-95-99.7 rule without justifying that the distribution of discoloration ratings in the
control group is approximately normal.

Part (b) is scored as essentially correct (E) if the student indicates that the preservative appears to be effective
and explicitly links this decision to comparison of a characteristic of relative standing from the dotplots for the
two groups.

Part (b) is scored as partially correct (P) if:


the student says that the preservative appears to be effective because the discoloration ratings appear to be
lower for the treatment group, but the student does not explicitly link this decision to comparison of a
characteristic of relative standing for the two groups;
OR
the student correctly compares one or more characteristics of relative standing for the two groups but never
states that the preservative was effective in lowering discoloration.

Part (b) is scored as incorrect (I) if:


the student says that the preservative is not effective because the centers of the two distributions are roughly
the same;
OR
the student says that the preservative is effective, with incorrect or no justification.

Part (c) is scored as essentially correct (E) if the student indicates that zero is not included in the confidence
interval, so there is a difference (in population means), AND states the conclusion in the context of this
experiment.

Part (c) is scored as partially correct (P) if:


the student indicates that zero is not included in the confidence interval, so there is a difference (in population
means), but does not state the conclusion in the context of this experiment;
OR
the student correctly interprets the 95 percent confidence interval in context and indicates that there is a
difference (in population means), without indicating that zero is not included in the confidence interval.

Part (c) is scored as incorrect (I) if the student concludes that the preservative is not effective OR says that no
conclusion can be made based on the confidence interval, OR the student states a conclusion that refers to sample
means instead of population means.

Notes:
• The student is not required to specify the significance level in part (c), but if it is specified, it must be
correct.

© 2007 The College Board. All rights reserved.


Visit apcentral.collegeboard.com (for AP professionals) and www.collegeboard.com/apstudents (for students and parents).
火箭学院 火速提分
新页码57

AP® STATISTICS
2007 SCORING GUIDELINES

Question 1 (continued)

• An adjustment could be made to formally conduct a one-sided test, but in general, confidence intervals
are used to conduct two-sided tests. The fact that the lower endpoint of the confidence interval is positive
does provide evidence that the preservative is effective in lowering the amount of discoloration in
strawberries. The correct formal statement is: The 97.5 percent lower confidence bound for the difference
in the means is above zero (0.16), so at the 0.025 level we would conclude that the mean rating for the
treated berries is significantly lower than the mean for the untreated strawberries.

4 Complete Response

All three parts essentially correct

3 Substantial Response

Two parts essentially correct and one part partially correct

2 Developing Response

Two parts essentially correct and no parts partially correct


OR
One part essentially correct and two parts partially correct
OR
Three parts partially correct

1 Minimal Response

One part essentially correct and either zero or one part partially correct
OR
No parts essentially correct and two parts partially correct

© 2007 The College Board. All rights reserved.


Visit apcentral.collegeboard.com (for AP professionals) and www.collegeboard.com/apstudents (for students and parents).
火箭学院 火速提分
新页码58

AP® STATISTICS
2007 SCORING GUIDELINES

Question 2

Intent of Question

The three primary goals of this question are to assess a student’s ability to: (1) clearly explain the importance of a
control group in the context of an experiment; (2) describe the randomization process required for three groups;
and (3) reduce variability by grouping experimental units as homogeneously as possible.

Solution

Part (a):

A control group gives the researchers a comparison group to be used to evaluate the effectiveness of the
treatments. The control group allows the impact of the normal aging process on joint and hip health to be
measured with appropriate response variables. The effects of glucosamine and chondroitin can be assessed by
comparing the responses for these two treatment groups with those for the control group.

Part (b):

Each dog will be assigned a unique random number, 001–300, using a random number generator on a
calculator, statistical software, or a random number table. The numbers will be sorted from smallest to largest.
The dogs assigned the first 100 numbers in the ordered list will receive glucosamine. The dogs with the next
100 numbers in the ordered list will be assigned to the control group. Finally, the dogs with the numbers 201–
300 will receive chondroitin.

Part (c):

The key question is which variable has the strongest association with joint and hip health. The goal of
blocking is to create groups of homogeneous experimental units. It is reasonable to assume that most clinics
will see all kinds and breeds of dogs so there is no reason to suspect that joint and hip health will be strongly
associated with a clinic. On the other hand, different breeds of dogs tend to come in different sizes. The size
of a dog is associated with joint and hip health, so it would be better to form homogeneous groups of dogs by
blocking on breed.

Scoring

Parts (a), (b), and (c) are scored as essentially correct (E), partially correct (P), or incorrect (I).

Part (a) is scored as essentially correct (E) if an advantage of using a comparison group is described in the
context of this study.

Part (a) is scored as partially correct (P) if an advantage of using a control group is described but not in the
context of this study.

Part (a) is scored as incorrect (I) if the student says that control groups should always be used but gives no further
explanation OR an incorrect explanation.

© 2007 The College Board. All rights reserved.


Visit apcentral.collegeboard.com (for AP professionals) and www.collegeboard.com/apstudents (for students and parents).
火箭学院 火速提分
新页码59

AP® STATISTICS
2007 SCORING GUIDELINES

Question 2 (continued)

Note: Since “treatment” and “control” are standard terms in design, a comparison of specific aspects of the study
is needed to establish context.

Part (b) is scored as essentially correct (E) if randomization is used correctly, and the method of randomization
can be implemented after reading the student response (so that two knowledgeable statistics users would use the
same method to assign dogs to treatment groups).

Part (b) is scored as partially correct (P) if randomization or chance is used, but the method could not be
implemented after reading the student response.

Part (b) is scored as incorrect (I) if randomization or chance is not used in a planned way OR the solution does not
yield a completely randomized design.

Part (c) is scored as essentially correct (E) if:


the student argues that the variable with the stronger relationship to joint and hip health should be used as the
blocking variable;
OR
the student states that the variable with the larger anticipated variability in the response measure should be
used as the blocking variable so that units within blocks are as homogeneous as possible. A rationale is
required, but a variable does not have to be selected.

Part (c) is scored as partially correct (P) if:


the student indicates that the purpose of blocking is to create groups of homogeneous experimental units but
makes an error in the application to this experiment;
OR
the student does not acknowledge that there may be more variability associated in the response variable with
one of the variables (breed or clinic) than the other;
OR
the student does not recognize that both variables are associated with variation in the response variable.

Part (c) is scored as incorrect (I) if the student does not exhibit an understanding of the purpose of blocking.

4 Complete Response

All three parts essentially correct

3 Substantial Response

Two parts essentially correct and one part partially correct

2 Developing Response

Two parts essentially correct and no parts partially correct


OR
One part essentially correct and two parts partially correct
OR
Three parts partially correct

© 2007 The College Board. All rights reserved.


Visit apcentral.collegeboard.com (for AP professionals) and www.collegeboard.com/apstudents (for students and parents).
火箭学院 火速提分
新页码60

AP® STATISTICS
2007 SCORING GUIDELINES

Question 2 (continued)

1 Minimal Response

One part essentially correct and either zero or one part partially correct
OR
No parts essentially correct and two parts partially correct

© 2007 The College Board. All rights reserved.


Visit apcentral.collegeboard.com (for AP professionals) and www.collegeboard.com/apstudents (for students and parents).
火箭学院 火速提分
新页码61

AP® STATISTICS
2007 SCORING GUIDELINES

Question 3

Intent of Question

This question was developed to assess a student’s understanding of the sampling distribution of the sample mean:
in particular, a student’s ability to: (1) compare probabilities concerning sample means from different sample
sizes; (2) compute an appropriate probability; and (3) recognize that an application of the Central Limit Theorem
is being evaluated.

Solution

Part (a):

The random sample of n = 15 fish is more likely to have a sample mean length greater than 10 inches. The
sampling distribution of the sample mean x is normal with mean m = 8 and standard deviation σ n .
Thus, both sampling distributions will be centered at 8 inches, but the sampling distribution of the sample
mean when n = 15 will have more variability than the sampling distribution of the sample mean when n = 50.
The tail area ( x >10) will be larger for the distribution that is less concentrated about the mean of 8 inches
when the sample size is n = 15, as shown in the following graph.

Variable
n=15
n=50

3 4 5 6 7 8 9 10 11 12
sample means

Part (b):
⎛ 7.5 − 8 ⎞ ⎛ 5⎞
P( x < 7.5) = P⎜ z < ⎟ = P⎜ z < − ⎟ = P( z < −1.67 ) = 0.0475
⎝ 0.3 ⎠ ⎝ 3⎠

prob = .0475

7.0 7.5 8.0 8.5 9.0


sample means

© 2007 The College Board. All rights reserved.


Visit apcentral.collegeboard.com (for AP professionals) and www.collegeboard.com/apstudents (for students and parents).
火箭学院 火速提分
新页码62

AP® STATISTICS
2007 SCORING GUIDELINES

Question 3 (continued)

Part (c):

Yes. The Central Limit Theorem says that the sampling distribution of the sample mean will become
approximately normal as the sample size n increases. Since the sample size is reasonably large
(n = 50), the calculation in part (b) will provide a good approximation to the probability of interest even
though the population is nonnormal.

Scoring

Parts (a), (b), and (c) are scored as essentially correct (E), partially correct (P), or incorrect (I).

Part (a) is scored as essentially correct (E) if the student says that the sample of 15 fish is more likely to have a
mean length that is greater than 10, AND the justification is based on variability in the sampling distributions.

Part (a) is scored as partially correct (P) if:


the student makes correct statements about the sampling distribution of the sample mean or the probabilities
but does not specifically refer to the variability in these two sampling distributions;
OR
the student remarks that the sample mean approaches the population mean as the sample size increases (an
argument based on the Law of Large Numbers).

Some examples of partially correct statements are:


• With the smaller sample size we will be more likely to get an extreme value for the sample mean.
• Variability in the smaller sample is larger.
• Variability in the larger sample is smaller.
• The sample mean approaches the population mean as the sample size increases.

Part (a) is scored as incorrect (I) if an answer is provided with no justification or incorrect justification.

Note: If a student chooses a particular value for a standard deviation and goes through the correct calculations,
or shows the result algebraically based on a generic standard deviation, then the response should be
scored essentially correct.

Part (b) is scored essentially correct (E) if the probability is calculated correctly and a reasonable sketch or
evidence of calculation is shown.

Part (b) is scored partially correct (P) if:


an incorrect but plausible calculation is shown. Examples include using an incorrect standard deviation (such
as 0.3 50 ) to obtain the probability;
OR
the student switches the sample mean and the population mean to obtain a standardized z value of 1.67.

© 2007 The College Board. All rights reserved.


Visit apcentral.collegeboard.com (for AP professionals) and www.collegeboard.com/apstudents (for students and parents).
火箭学院 火速提分
新页码63

AP® STATISTICS
2007 SCORING GUIDELINES

Question 3 (continued)

Part (b) is scored incorrect (I) if an answer is provided with no justification or incorrect justification.

Note: Normalcdf (…) with no additional work is at best partially correct. If an appropriate sketch accompanies
the calculator command, OR if the components of the calculator command are clearly identified/labeled,
then the solution should be scored essentially correct.

Part (c) is scored as essentially correct (E) if the student says that the probability is a reasonable approximation
because of the Central Limit Theorem and also refers to the large sample size in this case.

Part (c) is scored partially correct (P) if the student indicates that the response in part (b) would not change but
provides a weak justification. Examples of a weak justification include mentioning CLT without reference to
sample size, and mentioning sample size without reference to CLT.

Part (c) is scored incorrect (I) if an answer is provided with no justification or incorrect justification.

Note: An E counts for 2 points in part (a), and an E counts for 1 point in each of parts (b) and (c). Similarly, a
P counts for 1 point in part (a), and a P counts for ½ point in parts (b) and (c). When the total number of
points earned is not an integer, the final score earned will be rounded down to the integer value.

4 Complete Response

4 points earned

3 Substantial Response

3 or 3½ points earned

2 Developing Response

2 or 2½ points earned

1 Minimal Response

1 or 1½ points earned

© 2007 The College Board. All rights reserved.


Visit apcentral.collegeboard.com (for AP professionals) and www.collegeboard.com/apstudents (for students and parents).
火箭学院 火速提分
新页码64

AP® STATISTICS
2007 SCORING GUIDELINES

Question 4

Intent of Question

This statistical inference question was developed to assess a student’s ability to distinguish paired-data procedures
from two-sample procedures and to execute the selected procedure. The ability to provide a complete statistical
justification is an important skill that can be evaluated with this standard inference problem.

Solution

A hypothesis test for the mean difference in the level of E. coli bacteria contamination in beef detected by the two
methods will be conducted.

Part 1: States a correct pair of hypotheses:

H 0 : μd = 0
H a : μd ≠ 0
where μd is the mean difference (method A − method B) in the level of E. coli bacteria contamination in
beef detected by the two methods

Part 2: Identifies a correct test (by name or by formula) and checks appropriate conditions:

xd − 0
Paired t-test t =
sd nd

Conditions:
1. Since the observations are obtained on 10 randomly selected specimens, it is reasonable to assume that
the 10 data pairs are independent of one another.
2. The population distribution of differences is normal.

The computed differences are:


-0.3 0.5 0.3 0.6 0.8 0.7 1.2 0.2 -0.1 -1.0

Histogram of the differences (A-B):

This histogram of differences is symmetric with no apparent outliers. Even though it is hard to judge the overall
shape of a distribution with only 10 observations, it appears that the normal distribution is a reasonable option in
this case.

© 2007 The College Board. All rights reserved.


Visit apcentral.collegeboard.com (for AP professionals) and www.collegeboard.com/apstudents (for students and parents).
火箭学院 火速提分
新页码65

AP® STATISTICS
2007 SCORING GUIDELINES

Question 4 (continued)

Part 3: Correct mechanics, including the value of the test statistic, d.f., and P-value (or rejection region):

xd = 0.29 sd = 0.629727
0.29 − 0 0.29
t= = = 1.46 d.f. = 9 P-value = 0.179
0.629727 10 0.199137

OR

Calculator: t = 1.4563, P-value = 0.1793, d.f. = 9.

Part 4: States a correct conclusion in the context of the problem, using the result of the statistical test.

Since the P-value is greater than 0.05, we cannot reject H 0 . We do NOT have statistically significant
evidence to conclude that there is a difference in the mean amount of E. coli bacteria detected by the two
methods for this type of beef. In other words, there does not appear to be a significant difference in these two
methods for measuring the level of E. coli contamination in beef.

Scoring

Parts 1, 2, 3, and 4 are scored as essentially correct (E) or incorrect (I).

Part 1 is scored as essentially correct (E) if the student states a correct pair of hypotheses. The hypotheses may be
stated in terms of μ A and μ B . With any nonstandard notation used, the parameters must be identified in context
clearly indicating the population mean.

Part 2 is scored as essentially correct (E) if the student identifies a correct test (by name or formula) and checks
appropriate conditions. The conditions for the paired t-test are about the differences. If the student says that the
10 differences can be viewed as a SRS of all differences, the answer is acceptable. However, the student does not
need to repeat the fact that these specimens can be viewed as a random sample.

It is not acceptable to view all 20 observations as a random sample or two independent samples. If conditions are
stated and checked using the two separate samples, part 2 is scored as incorrect (I) for a paired t-test.

For part 2, a graphical check of normality is required. Graph(s) should be consistent with the data, AND students
must comment linking the graph to the condition.

Histogram of the differences (B-A):


This histogram of differences is roughly symmetric with no
apparent outliers. Even though it is hard to judge the overall
shape of a distribution with only 10 observations, it appears
that the normal distribution is a reasonable option in this case.

© 2007 The College Board. All rights reserved.


Visit apcentral.collegeboard.com (for AP professionals) and www.collegeboard.com/apstudents (for students and parents).
火箭学院 火速提分
新页码66

AP® STATISTICS
2007 SCORING GUIDELINES

Question 4 (continued)

Boxplot of differences (A-B): Boxplot of differences (B-A): The boxplot of the differences shows
that the distribution is approximately
symmetric with no outliers, so it is
reasonable to proceed with the paired
t-test.

Normal Probability Plot of Normal Probability Plot of


differences (A-B) differences (B-A) The normal probability plot shows linear
(data on x-axis): (data on x-axis): trend with no obvious departures from linear
trend, so the normal model is reasonable.

Part 3 is scored essentially correct (E) if the student performs correct mechanics when calculating the value of the
test statistic and correctly calculates the p-value for the rejection region.

Part 4 is scored essentially correct (E) if the student states a correct conclusion in the context of the problem,
using the result of the statistical test.

If the p-value in part 3 is incorrect but the conclusion is consistent with the computed p-value, part 4 can be
considered correct.

In part 4, if both an α and a p-value are given together, the linkage between the p-value and the conclusion is
implied.

If no α is given, the solution must be explicit about the linkage by giving a correct interpretation of the p-value
or explaining how the conclusion follows from the p-value.

Scoring Confidence Interval approach:


A confidence interval may be used to make the inference but must include all four parts to get full credit.

The confidence level must be stated to get credit for part 3.

A 95 percent confidence interval for μd is (-0.16, 0.74).

© 2007 The College Board. All rights reserved.


Visit apcentral.collegeboard.com (for AP professionals) and www.collegeboard.com/apstudents (for students and parents).
火箭学院 火速提分
新页码67

AP® STATISTICS
2007 SCORING GUIDELINES

Question 4 (continued)

Since zero is included in the 95 percent confidence interval, we cannot reject the null hypothesis at the
0.05 level. We do NOT have statistically significant evidence to conclude that there is a difference in the
mean amount of E.coli bacteria detected by the two methods for this type of beef. In other words, there does
not appear to be a significant difference in these two methods for measuring the level of E.coli contamination
in beef.

Scoring independent samples t-test or confidence interval approach:


If an independent samples t-test or confidence interval is done, the maximum score is 3, provided all
four parts for independent samples t-test are done correctly.

For the independent samples t-test or confidence interval, the condition of normality must be checked using
two samples separately.

t = 0.079 p = 0.9377 df = 18 (pooled) or 17.97 (unpooled)

A 95 percent independent-samples (two-sample) confidence interval for μ A − μ B is (-7.40, 7.98).

Each part is scored as correct or incorrect.

4 Complete Response

Four parts correct

3 Substantial Response

Three parts correct

2 Developing Response

Two parts correct

1 Minimal Response

One part correct

© 2007 The College Board. All rights reserved.


Visit apcentral.collegeboard.com (for AP professionals) and www.collegeboard.com/apstudents (for students and parents).
火箭学院 火速提分
新页码68

AP® STATISTICS
2007 SCORING GUIDELINES

Question 5

Intent of Question

The primary goals of this statistical inference question are to assess a student’s ability to: (1) distinguish an
observational study from an experiment; (2) state the appropriate hypotheses for a research problem;
(3) check the appropriate conditions for an inference procedure; and (4) interpret standard results for an inference
procedure that is unfamiliar to students.

Solution

Part (a):

This is an experiment because the researchers imposed treatments by randomly assigning drivers to the two
different conditions (simulated driving while talking on a cell phone versus simulated driving while talking to
a passenger).

Part (b):

Let pcell denote the proportion of drivers who miss an exit while using a cell phone and ppass denote the
proportion of drivers who miss an exit while talking to a passenger.

H0: pcell = ppass


HA: pcell > ppass

Part (c):

The conditions required for a two-sample z-test of equal proportions are:


(1) independent random samples or random assignment, and
(2) large sample sizes ⎡ n1 lp1 ≥ 10, n1 (1 − lp1 ) ≥ 10, n2 lp 2 ≥ 10, n2 (1 − lp 2 ) ≥ 10⎤ .
⎣ ⎦
Random assignment is stated in the stem so the first condition is met. However, the numbers of successes
( ncell lp cell = 7 and n pass lp pass = 2 ) are both smaller than 10, so the large sample condition is not met in this
situation. Note: If the student uses the rule of thumb with 10 replaced by 5, then the number of successes for
the second sample is still too small.

Part (d):

Interpretation: Assuming that talking on a cell phone and talking to a passenger are equally distracting (there
is no difference in the two population proportions of drivers who will miss the exit), the p-value measures the
chance of observing a difference in the two sample proportions as large as or larger than the one observed.

Conclusion: Since the p-value 0.0683 is larger than 0.05, we cannot reject the null hypothesis. That is, we do
not have statistically significant evidence to conclude that using a cell phone is more distracting to drivers
than talking to another passenger in the car.

Notice that if we increase the significance level to 0.1, then we could reject the null hypothesis and conclude
that drivers are significantly more distracted when using a cell phone.

© 2007 The College Board. All rights reserved.


Visit apcentral.collegeboard.com (for AP professionals) and www.collegeboard.com/apstudents (for students and parents).
火箭学院 火速提分
新页码69

AP® STATISTICS
2007 SCORING GUIDELINES

Question 5 (continued)

Scoring

Parts (a) and (b) are scored as essentially correct (E) or incorrect (I). Parts (c) and (d) are scored as essentially
correct (E), partially correct (P), or incorrect (I).

Part (a) is scored as essentially correct (E) if the student indicates that this is an experiment because treatments
were imposed.

Part (a) is scored as incorrect (I) if no explanation is provided, or the student says that this is an observational
study.

Part (b) is scored as essentially correct (E) if the student correctly identifies the two population proportions with
the correct hypotheses. Nonstandard notation must indicate reference to population proportions.

Part (b) is scored as incorrect (I) if the student is clearly referring to the sample proportions.

Part (c) is scored as essentially correct (E) if the student provides both conditions and correctly comments on
both.

Part (c) is scored as partially correct (P) if the student provides and correctly comments on only one of the
conditions.

Part (c) is scored as incorrect (I) if conditions are provided but no correct comments are given.

Part (d) is scored as essentially correct (E) if the p-value is correctly interpreted AND the correct conclusion is
provided AND context is given.

Part (d) is scored as partially correct (P) if:

i) either the p-value is correctly interpreted OR the correct conclusion is provided


AND
ii) context is given.

Part (d) is scored as incorrect (I) if neither a correct interpretation of the p-value in context NOR a correct
conclusion in context is provided.

In part (d) if both an α and a p-value are given together, the linkage between the p-value and the conclusion is
implied. If no α is given, the solution must be explicit about the linkage by giving a correct interpretation of the p-
value or explaining how the conclusion follows from the p-value.

Note: Any choice of an α could have been made as long as the appropriate interpretation is made relative to that
choice of α .

© 2007 The College Board. All rights reserved.


Visit apcentral.collegeboard.com (for AP professionals) and www.collegeboard.com/apstudents (for students and parents).
火箭学院 火速提分
新页码70

AP® STATISTICS
2007 SCORING GUIDELINES

Question 5 (continued)

Each essentially correct (E) response counts as 1 point; each partially correct (P) response counts as ½ point.

4 Complete Response

3 Substantial Response

2 Developing Response

1 Minimal Response

If a response is between two scores (for example, 2½ points), use a holistic approach to determine whether
to score up or down depending on the strength of the response and communication.

© 2007 The College Board. All rights reserved.


Visit apcentral.collegeboard.com (for AP professionals) and www.collegeboard.com/apstudents (for students and parents).
火箭学院 火速提分
新页码71

AP® STATISTICS
2007 SCORING GUIDELINES

Question 6

Intent of Question

This question was designed to evaluate a student’s ability to make inferences for simple linear regression models.
Interpreting model parameters and comparing and contrasting different models are important skills that are also
being assessed. Finally, a multiple regression model with a special variable, an indicator variable, is introduced to
investigate whether the relationship between the predictor and response variable differs for two different groups
of people. Students are asked to sketch the estimated line for both groups and interpret the estimated parameters in
the multiple regression model.

Solution

Part (a):

The value 1.080 estimates the average increase (in feet) in the perceived distance for each additional foot in
actual distance between the two objects.

Part (b):

The model with zero intercept makes more intuitive sense in this particular situation. If the two objects are
placed side by side (so the actual distance is zero), then we would expect the subjects to say that the distance
between the objects is zero.

Part (c):

Let β denote the true slope between the perceived distances and the actual distances. The researcher’s
hypothesis is equivalent to β > 1 . Thus, we want to conduct a hypothesis test for the slope parameter.

Step 1: States a correct pair of hypotheses:

H0 : β = 1
Ha : β > 1

Step 2: Correct mechanics, including the value of the test statistic and p-value (or rejection region).

This is a t-test of a slope.


b − β 1.102 − 1
t= = = 0.260
sb 0.393
df = 40 − 1 = 39
p-value = P(t > .260) = 0.398

© 2007 The College Board. All rights reserved.


Visit apcentral.collegeboard.com (for AP professionals) and www.collegeboard.com/apstudents (for students and parents).
火箭学院 火速提分
新页码72

AP® STATISTICS
2007 SCORING GUIDELINES

Question 6 (continued)

Step 3: States a correct conclusion in the context of the problem, using the result of the statistical test.

Since the p-value 0.398 is greater than 0.05, we cannot reject H 0 . That is, we do not have statistically
significant evidence to conclude that the subjects overestimate the distance with the magnitude of the
overestimation increasing as the actual distance increases.

Part (d):

According to Model 3, the estimated models for the two groups are:
Contact wearers (contact = 1):
perceived distance = 1.05 (actual distance) + 0.12 (actual distance)
= 1.17 (actual distance)
Noncontact wearers (contact = 0):
perceived distance = 1.05 (actual distance)

Contacts

No contacts

Part (e):

Model 3 allows prediction of perceived distance separately for contact wearers and for noncontact wearers.
The value of 1.05 estimates the average increase (in feet) in the perceived distance for each one-foot increase
in actual distance for the population of noncontact wearers. The value of 0.12 estimates the additional
increase (in feet) in the average perceived distance for each one-foot increase in actual distance for the contact
wearers.

© 2007 The College Board. All rights reserved.


Visit apcentral.collegeboard.com (for AP professionals) and www.collegeboard.com/apstudents (for students and parents).
火箭学院 火速提分
新页码73

AP® STATISTICS
2007 SCORING GUIDELINES

Question 6 (continued)

Scoring

Parts (a) and (b) are combined and scored as essentially correct (E), partially correct (P), or incorrect (I). Parts
(c), (d), and (e) are scored as essentially correct (E), partially correct (P), or incorrect (I).

Parts (a) and (b) combined is scored as essentially correct (E) if both parts are correct.

Parts (a) and (b) combined is scored as partially correct (P) if:
one part is correct and the other part is incorrect;
OR
one part is correct and the other part is partially correct;
OR
both parts are partially correct.

Part (a) and (b) combined is scored as incorrect (I) if one part is partially correct.

Notes:
Part (a) is scored as partially correct if there is no word that makes it clear that 1.080 is not a deterministic
increase.
Part (a) is scored as incorrect if the response:
• ignores the intercept and implies proportionality: for each foot of actual distance between the two
objects, the subject perceives about 1.080 feet;
• consists of the equation rewritten in words.

Part (b)
Additional correct statement:
• The intercept is clearly not statistically significant, so the simpler model that includes only the
slope is reasonable.
Partially correct statements:
• The SE for Model 2 is so large that Model 2 does not seem reasonable.
• The interpretation of the slope is straightforward if there is a 0 intercept: the percentage error is
slope – 1 or 10.2 percent.
• The slope for Model 2 is farther above 1 than the slope for Model 1 and so more in line with the
researcher’s hypothesis.

Incorrect statements:
• Having one SE is better than having two.
• It is simpler/easier/shorter/more accurate to have just one coefficient.

© 2007 The College Board. All rights reserved.


Visit apcentral.collegeboard.com (for AP professionals) and www.collegeboard.com/apstudents (for students and parents).
火箭学院 火速提分
新页码74

AP® STATISTICS
2007 SCORING GUIDELINES

Question 6 (continued)

Part (c) is scored as:

Essentially correct (E) if three steps are correct.

Partially correct (P) if two steps are correct.

Incorrect (I) if one step is correct.

Notes:
• Hypotheses: the hypotheses step is incorrect if the alternative hypothesis is two-sided, or if the null
hypothesis is β = 0. (It is not necessary to define β .)
• Computation: if the computation includes division by 40 , the computation step is incorrect.
• Conclusion: a conclusion with no context is incorrect.

Part (d) is scored as essentially correct (E) if both estimated regression lines are graphed correctly and at least
one is labeled.

Part (d) is scored as partially correct (P) if:


• the lines are graphed correctly but neither is labeled;
OR
• the graphs consist of unconnected dots.

Part (d) is scored as incorrect (I) if:


• the two lines on the grid have the same slope;
OR
• one line is plotted correctly and one line is not.

Part (e) is scored as essentially correct (E) if the response includes a correct interpretation of the estimated
coefficients, 1.05 and 0.12. Unlike in part (a) there is no y-intercept, so this statement is correct: “For each foot of
actual distance between the two objects, a noncontact wearer perceives about 1.05 feet, and a contact wearer will
perceive about an additional 0.12 feet.”

Part (e) is scored as partially correct (P) if:


• the response includes a correct interpretation of just one of the two coefficients;
OR
• the response includes a correct interpretation of 1.05 and 1.05 + 0.12 = 1.17 but doesn’t include a
separate interpretation of 0.12;
OR
• no numbers are mentioned, but it is made clear that both groups overestimate the distance AND that
contact wearers overestimate more than do noncontact wearers.

Part (e) is scored as incorrect (I) if:


• the response says only that 1.05 and 0.12 are “slopes of regression lines”;
OR
• only the SEs of the coefficients, 0.357 and 0.032, are interpreted.

© 2007 The College Board. All rights reserved.


Visit apcentral.collegeboard.com (for AP professionals) and www.collegeboard.com/apstudents (for students and parents).
火箭学院 火速提分
新页码75

AP® STATISTICS
2007 SCORING GUIDELINES
Question 6 (continued)

Each essentially correct (E) response counts as 1 point; each partially correct (P) response counts as ½ point.

4 Complete Response

3 Substantial Response

2 Developing Response

1 Minimal Response

If a response is between two scores (for example, 2½ points), use a holistic approach to determine whether
to score up or down depending on the strength of the response and communication.

© 2007 The College Board. All rights reserved.


Visit apcentral.collegeboard.com (for AP professionals) and www.collegeboard.com/apstudents (for students and parents).
火箭学院 火速提分
新页码76

AP® Statistics
2007 Free-Response Questions
Form B

The College Board: Connecting Students to College Success


The College Board is a not-for-profit membership association whose mission is to connect students to college success and
opportunity. Founded in 1900, the association is composed of more than 5,000 schools, colleges, universities, and other
educational organizations. Each year, the College Board serves seven million students and their parents, 23,000 high schools, and
3,500 colleges through major programs and services in college admissions, guidance, assessment, financial aid, enrollment, and
teaching and learning. Among its best-known programs are the SAT®, the PSAT/NMSQT®, and the Advanced Placement
® ®
Program (AP ). The College Board is committed to the principles of excellence and equity, and that commitment is embodied
in all of its programs, services, activities, and concerns.

© 2007 The College Board. All rights reserved. College Board, Advanced Placement Program, AP, AP Central, SAT, and the
acorn logo are registered trademarks of the College Board. PSAT/NMSQT is a registered trademark of the College Board and
National Merit Scholarship Corporation.
Permission to use copyrighted College Board materials may be requested online at:
www.collegeboard.com/inquiry/cbpermit.html.

Visit the College Board on the Web: www.collegeboard.com.


AP Central is the official online home for the AP Program: apcentral.collegeboard.com.
火箭学院 火速提分
新页码77
®
2007 AP STATISTICS FREE-RESPONSE QUESTIONS (Form B)

Formulas begin on page 3.


Questions begin on page 6.
Tables begin on page 13.

-2-
火箭学院 火速提分
新页码78
®
2007 AP STATISTICS FREE-RESPONSE QUESTIONS (Form B)

Formulas

(I) Descriptive Statistics

 xi
x =
n

( )
1 2
sx = Â xi - x
n -1

(n1 - 1)s21 + (n2 - 1)s22


sp =
(n1 - 1) + (n2 - 1)
ŷ = b0 + b1x

b1 =
( )(
 xi - x yi - y )
(
 xi - x )2
b0 = y - b1x

1 Êx - x ˆ Ê yi - y ˆ
r = ÂÁ i ˜¯ Á s ˜
n - 1 Ë sx Ë y ¯

sy
b1 = r
sx

(
 yi - yˆi )2
sb = n-2
1
(
 xi - x )2

-3-
火箭学院 火速提分
新页码79
®
2007 AP STATISTICS FREE-RESPONSE QUESTIONS (Form B)

(II) Probability

P ( A » B ) = P ( A) + P ( B ) - P ( A « B )

P ( A « B)
P ( A B) =
P ( B)

E ( X ) = μ x = Â xi pi

( )
2
Var( X ) = s 2x = Â xi - μ x pi

If X has a binomial distribution with parameters n and p, then:

Ê nˆ
P ( X = k ) = Á ˜ p k (1 - p)n - k
Ë k¯

μ x = np

s x = np(1 - p)

μ pˆ = p

p(1 - p)
s pˆ =
n

If x is the mean of a random sample of size n from an infinite


population with mean μ and standard deviation s , then:

μx = μ

s
sx =
n

-4-
火箭学院 火速提分
新页码80
®
2007 AP STATISTICS FREE-RESPONSE QUESTIONS (Form B)

(III) Inferential Statistics

statistic - parameter
Standardized test statistic:
standard deviation of statistic

Confidence interval: statistic ± (critical value ) ∑ (standard deviation of statistic )

Single-Sample

Standard Deviation
Statistic
of Statistic
s
Sample Mean n

p(1 - p)
Sample Proportion n

Two-Sample

Standard Deviation
Statistic
of Statistic

Difference of s12 s 22
sample means +
n1 n2

Special case when s1 = s 2


1 1
s +
n1 n2

Difference of p1 (1 - p1 ) p2 (1 - p2 )
sample proportions +
n1 n2

Special case when p1 = p2


1 1
p (1 - p ) +
n1 n2

(observed - expected )2
Chi-square test statistic = Â expected

-5-
火箭学院 火速提分
新页码81
®
2007 AP STATISTICS FREE-RESPONSE QUESTIONS (Form B)

STATISTICS
SECTION II
Part A
Questions 1-5
Spend about 65 minutes on this part of the exam.
Percent of Section II grade— 75

Directions: Show all your work. Indicate clearly the methods you use, because you will be graded on the
correctness of your methods as well as on the accuracy and completeness of your results and explanations.

1. The Better Business Council of a large city has concluded that students in the city’s schools are not learning
enough about economics to function in the modern world. These findings were based on test results from a
random sample of 20 twelfth-grade students who completed a 46-question multiple-choice test on basic
economic concepts. The data set below shows the number of questions that each of the 20 students in the
sample answered correctly.

12 16 18 17 18 33 41 44 38 35
19 36 19 13 43 8 16 14 10 9

(a) Display these data in a stemplot.


(b) Use your stemplot from part (a) to describe the main features of this score distribution.
(c) Why would it be misleading to report only a measure of center for this score distribution?

© 2007 The College Board. All rights reserved.


Visit apcentral.collegeboard.com (for AP professionals) and www.collegeboard.com/apstudents (for students and parents).

GO ON TO THE NEXT PAGE.


-6-
火箭学院 火速提分
新页码82
®
2007 AP STATISTICS FREE-RESPONSE QUESTIONS (Form B)

2. The graph below displays the relative frequency distribution for X, the total number of dogs and cats owned
per household, for the households in a large suburban area. For instance, 14 percent of the households own 2 of
these pets.

(a) According to a local law, each household in this area is prohibited from owning more than 3 of these pets. If
a household in this area is selected at random, what is the probability that the selected household will be in
violation of this law? Show your work.
(b) If 10 households in this area are selected at random, what is the probability that exactly 2 of them will be in
violation of this law? Show your work.
(c) The mean and standard deviation of X are 1.65 and 1.851, respectively. Suppose 150 households in this area
are to be selected at random and X , the mean number of dogs and cats per household, is to be computed.
Describe the sampling distribution of X , including its shape, center, and spread.

© 2007 The College Board. All rights reserved.


Visit apcentral.collegeboard.com (for AP professionals) and www.collegeboard.com/apstudents (for students and parents).

GO ON TO THE NEXT PAGE.


-7-
火箭学院 火速提分
新页码83
®
2007 AP STATISTICS FREE-RESPONSE QUESTIONS (Form B)

3. The United States Department of Energy is conducting an experiment to compare the heat gain in houses using
two different types of windows, A and B. Six windows of each type are available for the experiment. The
Department has constructed a house with twelve windows as shown on the floor plan below.

In the interior of the house, each window is surrounded by a window box to capture and measure the amount of
heat coming in through that window and to isolate the heat gain for each window.

(a) A randomized block experiment will be used to compare the heat gain for the two types (A and B) of
windows. How would you group the window boxes into blocks? (Clearly indicate your blocks using the
window box numbers.) Justify your choice of blocks.
(b) For the design in part (a), describe how you would assign window types (A and B) to the numbered window
boxes.

© 2007 The College Board. All rights reserved.


Visit apcentral.collegeboard.com (for AP professionals) and www.collegeboard.com/apstudents (for students and parents).

GO ON TO THE NEXT PAGE.


-8-
火箭学院 火速提分
新页码84
®
2007 AP STATISTICS FREE-RESPONSE QUESTIONS (Form B)

4. Each of 25 adult women was asked to provide her own height (y), in inches, and the height (x), in inches, of her
father. The scatterplot below displays the results. Only 22 of the 25 pairs are distinguishable because some of the
( x, y ) pairs were the same. The equation of the least squares regression line is yˆ = 35.1 + 0.427 x.

(a) Draw the least squares regression line on the scatterplot above.
(b) One father’s height was x = 67 inches and his daughter’s height was y = 61 inches. Circle the point on the
scatterplot above that represents this pair and draw the segment on the scatterplot that corresponds to the
residual for it. Give a numerical value for the residual.
(c) Suppose the point x = 84 , y = 71 is added to the data set. Would the slope of the least squares regression
line increase, decrease, or remain about the same? Explain.
(Note: No calculations are necessary to answer this question.)

Would the correlation increase, decrease, or remain about the same? Explain.
(Note: No calculations are necessary to answer this question.)

© 2007 The College Board. All rights reserved.


Visit apcentral.collegeboard.com (for AP professionals) and www.collegeboard.com/apstudents (for students and parents).

GO ON TO THE NEXT PAGE.


-9-
火箭学院 火速提分
新页码85
®
2007 AP STATISTICS FREE-RESPONSE QUESTIONS (Form B)

5. A serum cholesterol level above 250 milligrams per deciliter (mg/dl) of blood is a risk factor for cardiovascular
disease in humans. At a medical center in St. Louis, a study to test the effectiveness of a new cholesterol-
lowering drug was conducted. One hundred people with cholesterol levels between 250 mg/dl and 300 mg/dl
were available for this study. Fifty people were assigned at random to each of two treatment groups. One group
received the standard cholesterol-lowering medication and the other group received the new drug. After taking
the drug for three weeks, the 50 subjects who received the standard treatment had a mean decrease in cholesterol
level of 10 mg/dl with a standard deviation of 8 mg/dl, and the 50 subjects who received the new drug had a
mean decrease of 18 mg/dl with a standard deviation of 12 mg/dl.
Does the new drug appear to be more effective than the standard treatment in lowering mean cholesterol level?
Give appropriate statistical evidence to support your conclusion.

© 2007 The College Board. All rights reserved.


Visit apcentral.collegeboard.com (for AP professionals) and www.collegeboard.com/apstudents (for students and parents).

GO ON TO THE NEXT PAGE.


-10-
火箭学院 火速提分
新页码86
®
2007 AP STATISTICS FREE-RESPONSE QUESTIONS (Form B)

STATISTICS
SECTION II
Part B
Question 6
Spend about 25 minutes on this part of the exam.
Percent of Section II grade— 25

Directions: Show all your work. Indicate clearly the methods you use, because you will be graded on the
correctness of your methods as well as on the accuracy and completeness of your results and explanations.

6. Scientists interested in preserving natural habitats and minimizing the possible extinction of certain bird species
conducted a study to determine if it is better for conservation groups to purchase a few large nature preserves or
many small preserves in order to meet these goals.
The scientists studied 13 randomly selected islands of different sizes to determine the risk of extinction for bird
species. Islands are thought to be a good imitation of what would happen in a nature preserve because of their
isolation. If a species lived on only one island, it was considered to be at risk. Scientists have determined that
whether or not one species becomes extinct is independent of whether or not another species becomes extinct.
In 1990 scientists counted the number of at-risk species on each of the selected islands. They returned to each of
these islands in the year 2000 to see whether the species still existed on the islands. Species that were present in
1990 but absent in 2000 were considered extinct. Data collected by the scientists are given in the table below.

Island Area Species at Risk Species Extinct Proportion


(in sq km) in 1990 by 2000 Extinct
1 46 75 8 0.11
2 36 67 3 0.04
3 31 66 8 0.12
4 9 51 8 0.16
5 5 28 5 0.18
6 5 20 6 0.30
7 4 43 10 0.23
8 4 31 5 0.16
9 3 28 7 0.25
10 2 32 8 0.25
11 1 30 8 0.27
12 1 20 4 0.20
13 1 16 5 0.31

© 2007 The College Board. All rights reserved.


Visit apcentral.collegeboard.com (for AP professionals) and www.collegeboard.com/apstudents (for students and parents).

GO ON TO THE NEXT PAGE.


-11-
火箭学院 火速提分
新页码87
®
2007 AP STATISTICS FREE-RESPONSE QUESTIONS (Form B)

(a) One scientist involved in the study believes that large islands (those with areas greater than 25 square
kilometers) are more effective than small islands (those with areas of no more than 25 square kilometers)
for protecting at-risk species. The scientist noted that for this study, a total of 19 of the 208 species on the
large islands became extinct, whereas a total of 66 of the 299 species on the small islands became extinct.
Assume that the probability of extinction is the same for all at-risk species on large islands and the same
for all at-risk species on small islands. Do these data support the scientist's belief? Give appropriate
statistical justification for your answer.
(b) Another scientist who worked on this study thinks that the proportion of species that become extinct is
more directly related to the size of the islands than simply to whether the islands are grouped as large or
small. This scientist investigated the relationship between the proportion of extinct birds and the area, in
square kilometers, of islands. A least squares analysis was conducted on the proportion extinct and ln(area).
The regression analysis output, the scatterplot, and the residual plot are shown below.

Predictor Coef StDev T P


Constant 0.28996 0.01269 22.85 0.000
ln(area) -0.05323 0.00618 -8.61 0.000
S = 0.02863 R-Sq = 87.1%

Estimate the slope of the least squares regression line using a 95 percent confidence interval. Interpret your
answer in the context of this situation.
(c) In part (a), the scientist assumed that the probability of a species becoming extinct is the same for each of the
large islands. Similarly, the scientist assumed that the probability is the same for each of the small islands.
Based on your answer in part (b), do you think this is a reasonable assumption? Explain.
(d) A conservation group with a long-term goal of preserving species believes that all at-risk species will
disappear whenever land inhabited by those species is developed. It has an opportunity to purchase land in
an area about to be developed. The group has a choice of creating one large nature preserve with an area
of 45 square kilometers and containing 70 at-risk species, or 5 small nature preserves, each with an area
of 3 square kilometers and each containing 16 at-risk species unique to that preserve. Which choice would
you recommend and why?

STOP

END OF EXAM

© 2007 The College Board. All rights reserved.


Visit apcentral.collegeboard.com (for AP professionals) and www.collegeboard.com/apstudents (for students and parents).

-12-
火箭学院 火速提分
新页码88
®
2007 AP STATISTICS FREE-RESPONSE QUESTIONS (Form B)

Probability

Table entry for z is the


probability lying below z.
z
Table A Standard normal probabilities

z .00 .01 .02 .03 .04 .05 .06 .07 .08 .09
– 3.4 .0003 .0003 .0003 .0003 .0003 .0003 .0003 .0003 .0003 .0002
– 3.3 .0005 .0005 .0005 .0004 .0004 .0004 .0004 .0004 .0004 .0003
– 3.2 .0007 .0007 .0006 .0006 .0006 .0006 .0006 .0005 .0005 .0005
– 3.1 .0010 .0009 .0009 .0009 .0008 .0008 .0008 .0008 .0007 .0007
– 3.0 .0013 .0013 .0013 .0012 .0012 .0011 .0011 .0011 .0010 .0010
– 2.9 .0019 .0018 .0018 .0017 .0016 .0016 .0015 .0015 .0014 .0014
– 2.8 .0026 .0025 .0024 .0023 .0023 .0022 .0021 .0021 .0020 .0019
– 2.7 .0035 .0034 .0033 .0032 .0031 .0030 .0029 .0028 .0027 .0026
– 2.6 .0047 .0045 .0044 .0043 .0041 .0040 .0039 .0038 .0037 .0036
– 2.5 .0062 .0060 .0059 .0057 .0055 .0054 .0052 .0051 .0049 .0048
– 2.4 .0082 .0080 .0078 .0075 .0073 .0071 .0069 .0068 .0066 .0064
– 2.3 .0107 .0104 .0102 .0099 .0096 .0094 .0091 .0089 .0087 .0084
– 2.2 .0139 .0136 .0132 .0129 .0125 .0122 .0119 .0116 .0113 .0110
– 2.1 .0179 .0174 .0170 .0166 .0162 .0158 .0154 .0150 .0146 .0143
– 2.0 .0228 .0222 .0217 .0212 .0207 .0202 .0197 .0192 .0188 .0183
– 1.9 .0287 .0281 .0274 .0268 .0262 .0256 .0250 .0244 .0239 .0233
– 1.8 .0359 .0351 .0344 .0336 .0329 .0322 .0314 .0307 .0301 .0294
– 1.7 .0446 .0436 .0427 .0418 .0409 .0401 .0392 .0384 .0375 .0367
– 1.6 .0548 .0537 .0526 .0516 .0505 .0495 .0485 .0475 .0465 .0455
– 1.5 .0668 .0655 .0643 .0630 .0618 .0606 .0594 .0582 .0571 .0559
– 1.4 .0808 .0793 .0778 .0764 .0749 .0735 .0721 .0708 .0694 .0681
– 1.3 .0968 .0951 .0934 .0918 .0901 .0885 .0869 .0853 .0838 .0823
– 1.2 .1151 .1131 .1112 .1093 .1075 .1056 .1038 .1020 .1003 .0985
– 1.1 .1357 .1335 .1314 .1292 .1271 .1251 .1230 .1210 .1190 .1170
– 1.0 .1587 .1562 .1539 .1515 .1492 .1469 .1446 .1423 .1401 .1379
– 0.9 .1841 .1814 .1788 .1762 .1736 .1711 .1685 .1660 .1635 .1611
– 0.8 .2119 .2090 .2061 .2033 .2005 .1977 .1949 .1922 .1894 .1867
– 0.7 .2420 .2389 .2358 .2327 .2296 .2266 .2236 .2206 .2177 .2148
– 0.6 .2743 .2709 .2676 .2643 .2611 .2578 .2546 .2514 .2483 .2451
– 0.5 .3085 .3050 .3015 .2981 .2946 .2912 .2877 .2843 .2810 .2776
– 0.4 .3446 .3409 .3372 .3336 .3300 .3264 .3228 .3192 .3156 .3121
– 0.3 .3821 .3783 .3745 .3707 .3669 .3632 .3594 .3557 .3520 .3483
– 0.2 .4207 .4168 .4129 .4090 .4052 .4013 .3974 .3936 .3897 .3859
– 0.1 .4602 .4562 .4522 .4483 .4443 .4404 .4364 .4325 .4286 .4247
– 0.0 .5000 .4960 .4920 .4880 .4840 .4801 .4761 .4721 .4681 .4641

-13-
火箭学院 火速提分
新页码89
®
2007 AP STATISTICS FREE-RESPONSE QUESTIONS (Form B)

Probability

Table entry for z is the


probability lying below z.
z
Table A (Continued)

z .00 .01 .02 .03 .04 .05 .06 .07 .08 .09
0.0 .5000 .5040 .5080 .5120 .5160 .5199 .5239 .5279 .5319 .5359
0.1 .5398 .5438 .5478 .5517 .5557 .5596 .5636 .5675 .5714 .5753
0.2 .5793 .5832 .5871 .5910 .5948 .5987 .6026 .6064 .6103 .6141
0.3 .6179 .6217 .6255 .6293 .6331 .6368 .6406 .6443 .6480 .6517
0.4 .6554 .6591 .6628 .6664 .6700 .6736 .6772 .6808 .6844 .6879
0.5 .6915 .6950 .6985 .7019 .7054 .7088 .7123 .7157 .7190 .7224
0.6 .7257 .7291 .7324 .7357 .7389 .7422 .7454 .7486 .7517 .7549
0.7 .7580 .7611 .7642 .7673 .7704 .7734 .7764 .7794 .7823 .7852
0.8 .7881 .7910 .7939 .7967 .7995 .8023 .8051 .8078 .8106 .8133
0.9 .8159 .8186 .8212 .8238 .8264 .8289 .8315 .8340 .8365 .8389
1.0 .8413 .8438 .8461 .8485 .8508 .8531 .8554 .8577 .8599 .8621
1.1 .8643 .8665 .8686 .8708 .8729 .8749 .8770 .8790 .8810 .8830
1.2 .8849 .8869 .8888 .8907 .8925 .8944 .8962 .8980 .8997 .9015
1.3 .9032 .9049 .9066 .9082 .9099 .9115 .9131 .9147 .9162 .9177
1.4 .9192 .9207 .9222 .9236 .9251 .9265 .9279 .9292 .9306 .9319
1.5 .9332 .9345 .9357 .9370 .9382 .9394 .9406 .9418 .9429 .9441
1.6 .9452 .9463 .9474 .9484 .9495 .9505 .9515 .9525 .9535 .9545
1.7 .9554 .9564 .9573 .9582 .9591 .9599 .9608 .9616 .9625 .9633
1.8 .9641 .9649 .9656 .9664 .9671 .9678 .9686 .9693 .9699 .9706
1.9 .9713 .9719 .9726 .9732 .9738 .9744 .9750 .9756 .9761 .9767
2.0 .9772 .9778 .9783 .9788 .9793 .9798 .9803 .9808 .9812 .9817
2.1 .9821 .9826 .9830 .9834 .9838 .9842 .9846 .9850 .9854 .9857
2.2 .9861 .9864 .9868 .9871 .9875 .9878 .9881 .9884 .9887 .9890
2.3 .9893 .9896 .9898 .9901 .9904 .9906 .9909 .9911 .9913 .9916
2.4 .9918 .9920 .9922 .9925 .9927 .9929 .9931 .9932 .9934 .9936
2.5 .9938 .9940 .9941 .9943 .9945 .9946 .9948 .9949 .9951 .9952
2.6 .9953 .9955 .9956 .9957 .9959 .9960 .9961 .9962 .9963 .9964
2.7 .9965 .9966 .9967 .9968 .9969 .9970 .9971 .9972 .9973 .9974
2.8 .9974 .9975 .9976 .9977 .9977 .9978 .9979 .9979 .9980 .9981
2.9 .9981 .9982 .9982 .9983 .9984 .9984 .9985 .9985 .9986 .9986
3.0 .9987 .9987 .9987 .9988 .9988 .9989 .9989 .9989 .9990 .9990
3.1 .9990 .9991 .9991 .9991 .9992 .9992 .9992 .9992 .9993 .9993
3.2 .9993 .9993 .9994 .9994 .9994 .9994 .9994 .9995 .9995 .9995
3.3 .9995 .9995 .9995 .9996 .9996 .9996 .9996 .9996 .9996 .9997
3.4 .9997 .9997 .9997 .9997 .9997 .9997 .9997 .9997 .9997 .9998

-14-
火箭学院 火速提分
新页码90
®
2007 AP STATISTICS FREE-RESPONSE QUESTIONS (Form B)

Table entry for p and


C is the point t* with Probability p
probability p lying
above it and
probability C lying
between −t * and t*.

t*

Table B t distribution critical values

Tail probability p
df .25 .20 .15 .10 .05 .025 .02 .01 .005 .0025 .001 .0005
1 1.000 1.376 1.963 3.078 6.314 12.71 15.89 31.82 63.66 127.3 318.3 636.6
2 .816 1.061 1.386 1.886 2.920 4.303 4.849 6.965 9.925 14.09 22.33 31.60
3 .765 .978 1.250 1.638 2.353 3.182 3.482 4.541 5.841 7.453 10.21 12.92
4 .741 .941 1.190 1.533 2.132 2.776 2.999 3.747 4.604 5.598 7.173 8.610
5 .727 .920 1.156 1.476 2.015 2.571 2.757 3.365 4.032 4.773 5.893 6.869
6 .718 .906 1.134 1.440 1.943 2.447 2.612 3.143 3.707 4.317 5.208 5.959
7 .711 .896 1.119 1.415 1.895 2.365 2.517 2.998 3.499 4.029 4.785 5.408
8 .706 .889 1.108 1.397 1.860 2.306 2.449 2.896 3.355 3.833 4.501 5.041
9 .703 .883 1.100 1.383 1.833 2.262 2.398 2.821 3.250 3.690 4.297 4.781
10 .700 .879 1.093 1.372 1.812 2.228 2.359 2.764 3.169 3.581 4.144 4.587
11 .697 .876 1.088 1.363 1.796 2.201 2.328 2.718 3.106 3.497 4.025 4.437
12 .695 .873 1.083 1.356 1.782 2.179 2.303 2.681 3.055 3.428 3.930 4.318
13 .694 .870 1.079 1.350 1.771 2.160 2.282 2.650 3.012 3.372 3.852 4.221
14 .692 .868 1.076 1.345 1.761 2.145 2.264 2.624 2.977 3.326 3.787 4.140
15 .691 .866 1.074 1.341 1.753 2.131 2.249 2.602 2.947 3.286 3.733 4.073
16 .690 .865 1.071 1.337 1.746 2.120 2.235 2.583 2.921 3.252 3.686 4.015
17 .689 .863 1.069 1.333 1.740 2.110 2.224 2.567 2.898 3.222 3.646 3.965
18 .688 .862 1.067 1.330 1.734 2.101 2.214 2.552 2.878 3.197 3.611 3.922
19 .688 .861 1.066 1.328 1.729 2.093 2.205 2.539 2.861 3.174 3.579 3.883
20 .687 .860 1.064 1.325 1.725 2.086 2.197 2.528 2.845 3.153 3.552 3.850
21 .686 .859 1.063 1.323 1.721 2.080 2.189 2.518 2.831 3.135 3.527 3.819
22 .686 .858 1.061 1.321 1.717 2.074 2.183 2.508 2.819 3.119 3.505 3.792
23 .685 .858 1.060 1.319 1.714 2.069 2.177 2.500 2.807 3.104 3.485 3.768
24 .685 .857 1.059 1.318 1.711 2.064 2.172 2.492 2.797 3.091 3.467 3.745
25 .684 .856 1.058 1.316 1.708 2.060 2.167 2.485 2.787 3.078 3.450 3.725
26 .684 .856 1.058 1.315 1.706 2.056 2.162 2.479 2.779 3.067 3.435 3.707
27 .684 .855 1.057 1.314 1.703 2.052 2.158 2.473 2.771 3.057 3.421 3.690
28 .683 .855 1.056 1.313 1.701 2.048 2.154 2.467 2.763 3.047 3.408 3.674
29 .683 .854 1.055 1.311 1.699 2.045 2.150 2.462 2.756 3.038 3.396 3.659
30 .683 .854 1.055 1.310 1.697 2.042 2.147 2.457 2.750 3.030 3.385 3.646
40 .681 .851 1.050 1.303 1.684 2.021 2.123 2.423 2.704 2.971 3.307 3.551
50 .679 .849 1.047 1.299 1.676 2.009 2.109 2.403 2.678 2.937 3.261 3.496
60 .679 .848 1.045 1.296 1.671 2.000 2.099 2.390 2.660 2.915 3.232 3.460
80 .678 .846 1.043 1.292 1.664 1.990 2.088 2.374 2.639 2.887 3.195 3.416
100 .677 .845 1.042 1.290 1.660 1.984 2.081 2.364 2.626 2.871 3.174 3.390
1000 .675 .842 1.037 1.282 1.646 1.962 2.056 2.330 2.581 2.813 3.098 3.300
⬁ .674 .841 1.036 1.282 1.645 1.960 2.054 2.326 2.576 2.807 3.091 3.291

50% 60% 70% 80% 90% 95% 96% 98% 99% 99.5% 99.8% 99.9%
Confidence level C

-15-
火箭学院 火速提分
新页码91
®
2007 AP STATISTICS FREE-RESPONSE QUESTIONS (Form B)

Probability p
Table entry for p is the point
( χ 2 ) with probability p lying
above it.

(χ2 )

Table C c 2 critical values


Tail probability p
df .25 .20 .15 .10 .05 .025 .02 .01 .005 .0025 .001 .0005
1 1.32 1.64 2.07 2.71 3.84 5.02 5.41 6.63 7.88 9.14 10.83 12.12
2 2.77 3.22 3.79 4.61 5.99 7.38 7.82 9.21 10.60 11.98 13.82 15.20
3 4.11 4.64 5.32 6.25 7.81 9.35 9.84 11.34 12.84 14.32 16.27 17.73
4 5.39 5.99 6.74 7.78 9.49 11.14 11.67 13.28 14.86 16.42 18.47 20.00
5 6.63 7.29 8.12 9.24 11.07 12.83 13.39 15.09 16.75 18.39 20.51 22.11
6 7.84 8.56 9.45 10.64 12.59 14.45 15.03 16.81 18.55 20.25 22.46 24.10
7 9.04 9.80 10.75 12.02 14.07 16.01 16.62 18.48 20.28 22.04 24.32 26.02
8 10.22 11.03 12.03 13.36 15.51 17.53 18.17 20.09 21.95 23.77 26.12 27.87
9 11.39 12.24 13.29 14.68 16.92 19.02 19.68 21.67 23.59 25.46 27.88 29.67
10 12.55 13.44 14.53 15.99 18.31 20.48 21.16 23.21 25.19 27.11 29.59 31.42
11 13.70 14.63 15.77 17.28 19.68 21.92 22.62 24.72 26.76 28.73 31.26 33.14
12 14.85 15.81 16.99 18.55 21.03 23.34 24.05 26.22 28.30 30.32 32.91 34.82
13 15.98 16.98 18.20 19.81 22.36 24.74 25.47 27.69 29.82 31.88 34.53 36.48
14 17.12 18.15 19.41 21.06 23.68 26.12 26.87 29.14 31.32 33.43 36.12 38.11
15 18.25 19.31 20.60 22.31 25.00 27.49 28.26 30.58 32.80 34.95 37.70 39.72
16 19.37 20.47 21.79 23.54 26.30 28.85 29.63 32.00 34.27 36.46 39.25 41.31
17 20.49 21.61 22.98 24.77 27.59 30.19 31.00 33.41 35.72 37.95 40.79 42.88
18 21.60 22.76 24.16 25.99 28.87 31.53 32.35 34.81 37.16 39.42 42.31 44.43
19 22.72 23.90 25.33 27.20 30.14 32.85 33.69 36.19 38.58 40.88 43.82 45.97
20 23.83 25.04 26.50 28.41 31.41 34.17 35.02 37.57 40.00 42.34 45.31 47.50
21 24.93 26.17 27.66 29.62 32.67 35.48 36.34 38.93 41.40 43.78 46.80 49.01
22 26.04 27.30 28.82 30.81 33.92 36.78 37.66 40.29 42.80 45.20 48.27 50.51
23 27.14 28.43 29.98 32.01 35.17 38.08 38.97 41.64 44.18 46.62 49.73 52.00
24 28.24 29.55 31.13 33.20 36.42 39.36 40.27 42.98 45.56 48.03 51.18 53.48
25 29.34 30.68 32.28 34.38 37.65 40.65 41.57 44.31 46.93 49.44 52.62 54.95
26 30.43 31.79 33.43 35.56 38.89 41.92 42.86 45.64 48.29 50.83 54.05 56.41
27 31.53 32.91 34.57 36.74 40.11 43.19 44.14 46.96 49.64 52.22 55.48 57.86
28 32.62 34.03 35.71 37.92 41.34 44.46 45.42 48.28 50.99 53.59 56.89 59.30
29 33.71 35.14 36.85 39.09 42.56 45.72 46.69 49.59 52.34 54.97 58.30 60.73
30 34.80 36.25 37.99 40.26 43.77 46.98 47.96 50.89 53.67 56.33 59.70 62.16
40 45.62 47.27 49.24 51.81 55.76 59.34 60.44 63.69 66.77 69.70 73.40 76.09
50 56.33 58.16 60.35 63.17 67.50 71.42 72.61 76.15 79.49 82.66 86.66 89.56
60 66.98 68.97 71.34 74.40 79.08 83.30 84.58 88.38 91.95 95.34 99.61 102.7
80 88.13 90.41 93.11 96.58 101.9 106.6 108.1 112.3 116.3 120.1 124.8 128.3
100 109.1 111.7 114.7 118.5 124.3 129.6 131.1 135.8 140.2 144.3 149.4 153.2

-16-
火箭学院 火速提分
新页码92

AP® Statistics
2007 Scoring Guidelines
Form B

The College Board: Connecting Students to College Success


The College Board is a not-for-profit membership association whose mission is to connect students to college success and
opportunity. Founded in 1900, the association is composed of more than 5,000 schools, colleges, universities, and other
educational organizations. Each year, the College Board serves seven million students and their parents, 23,000 high schools, and
3,500 colleges through major programs and services in college admissions, guidance, assessment, financial aid, enrollment, and
teaching and learning. Among its best-known programs are the SAT®, the PSAT/NMSQT®, and the Advanced Placement
® ®
Program (AP ). The College Board is committed to the principles of excellence and equity, and that commitment is embodied
in all of its programs, services, activities, and concerns.

© 2007 The College Board. All rights reserved. College Board, Advanced Placement Program, AP, AP Central, SAT, and the
acorn logo are registered trademarks of the College Board. PSAT/NMSQT is a registered trademark of the College Board and
National Merit Scholarship Corporation.
Permission to use copyrighted College Board materials may be requested online at:
www.collegeboard.com/inquiry/cbpermit.html.

Visit the College Board on the Web: www.collegeboard.com.


AP Central is the official online home for the AP Program: apcentral.collegeboard.com.
火箭学院 火速提分
新页码93

AP® STATISTICS
2007 SCORING GUIDELINES (Form B)

Question 1

Intent of Question

The three primary goals of this question are to assess a student’s ability to: (1) construct a stemplot from a given
data set; (2) describe the important features of the plot; and (3) discuss how a single measure of centrality fails to
convey important features of the plot.

Solution

Part (a):

0| 89
1| 26878993640
2|
3| 3856
4| 143 Legend: 1| 2 represents 12 questions answered correctly

OR, with ordered leaves (not required)

0| 89
1| 02346678899
2|
3| 3568
4| 134 Legend: 1| 2 represents 12 questions answered correctly

OR, with repeated stems (leaves may be ordered or not)

0H 89
1L 0234
1H 6678899
2L
2H
3L 3
3H 568 Legend: 3H 6 represents 36 questions answered correctly
4L 134 4L 1 represents 41 questions answered correctly

Part (b):

The most striking feature of the plot is that the scores cluster into two groups, one concentrated in the
mid-teens and the other in the high 30s (or one with relatively low scores on the exam and one with
relatively high scores). There are no scores in the 20s.

Part (c):

A measure of center might fall between the two groups (as does the mean of 22.95 here) where there is no
data and would not provide an accurate picture of student performance on the exam. It would not indicate
that students tended to score either very well or very poorly on the exam.

© 2007 The College Board. All rights reserved.


Visit apcentral.collegeboard.com (for AP professionals) and www.collegeboard.com/apstudents (for students and parents).
火箭学院 火速提分
新页码94

AP® STATISTICS
2007 SCORING GUIDELINES (Form B)

Question 1 (continued)

Scoring

This question is scored in four sections: section 1 is part (a), and sections 2 to 4 consist of elements of parts (b)
and (c).

Section 1 is scored as either essentially correct (E) or incorrect (I).

Section 1 is essentially correct (E) if in part (a) the student gives a correctly constructed stemplot. Any other type
of plot is incorrect (I).

NOTE: One or two misplaced or omitted leaves can still be considered essentially correct as long as the important
features of the display are not altered.

Parts (b) and (c) are scored together in three sections, each of which is scored as essentially correct (E), partially
correct (P), or incorrect (I).

Section 2 is essentially correct (E) if in either part (b) or (c) the student clearly notices:
1. that there are two groups;
2. that there is a gap in the middle of the distribution;
3. the relative or specific positions of the two groups,
OR
the location of the gap,
OR
a general measure of location (such as mean, median, or the fact that most scores fall between 10 and
19). (Median = 18, mean = 22.95)

Section 2 is partially correct (P) if the student notes two out of the three.

Section 3 is essentially correct (E) if in part (b) or part (c) the solution is given in the context of the problem and
is communicated well.

Section 3 is partially correct (P) if the student mentions the context (for instance, using the word “scores”), but
communication of the context is weak.

Section 3 is incorrect (I) if the context is not mentioned at all.

Section 4 is essentially correct (E) if in part (c) a valid reason is given for why a measure of center is not
sufficient for data of this type (with the two groups and a gap). If, for instance, the reasoning would apply equally
well to other shapes, it is not sufficient.

Section 4 can be at most partially correct (P) if a student does not recognize the groups or gap. It is partially
correct if the student compares the mean and median and cites outliers or skewness as the reason why a measure
of center is not sufficient, or if a general reason is given for why a measure of center is not sufficient. (For
instance, the student may say that center alone without some measure of spread is never sufficient.)

© 2007 The College Board. All rights reserved.


Visit apcentral.collegeboard.com (for AP professionals) and www.collegeboard.com/apstudents (for students and parents).
火箭学院 火速提分
新页码95

AP® STATISTICS
2007 SCORING GUIDELINES (Form B)

Question 1 (continued)

4 Complete Response

All four sections essentially correct

3 Substantial Response

Three sections essentially correct and no sections partially correct


OR
Two sections essentially correct and two sections partially correct

2 Developing Response

Two sections essentially correct and no sections partially correct


OR
One section essentially correct and two sections partially correct

Note: A score cannot exceed 2 if (1) the student fails to notice either of the two distinct groups of scores
or the gap between the groups, and (2) the response to part (c) mentions neither the two groups nor the
gap.

1 Minimal Response

One section essentially correct and no sections partially correct


OR
No sections essentially correct and two sections partially correct

If a response is between two scores (for example, 2½ points), use a holistic approach to determine whether
to score up or down depending on the strength of the response and communication.

© 2007 The College Board. All rights reserved.


Visit apcentral.collegeboard.com (for AP professionals) and www.collegeboard.com/apstudents (for students and parents).
火箭学院 火速提分
新页码96

AP® STATISTICS
2007 SCORING GUIDELINES (Form B)

Question 2

Intent of Question

The three primary goals of this question are to assess a student’s ability to: (1) calculate a probability from a
display of population frequencies; (2) calculate a binomial probability; and (3) describe a sampling distribution of
a sample mean for a moderately large sample.

Solution

Part (a):

P( X > 3) = 0.07 + 0.04 + 0.04 + 0.02 = 0.17.

Part (b):

Y = number of households in violation.

Y has a binomial distribution with n = 10 and p = 0.17.

⎛10 ⎞
P (Y = 2) = ⎜ ⎟ (0.17) 2 (0.83)8 = 0.2929.
⎝2⎠
Part (c):

The distribution of X will:


1. be approximately normal;
2. have mean μ X = μ = 1.65;
σ 1.851
3. have standard deviation σ X = = = 0.1511.
n 150

Scoring

This question is scored in four sections. Each section is scored as either essentially correct (E), partially correct
(P), or incorrect (I).

Section 1 is part (a), section 2 is part (b), and sections 3 and 4 consist of elements of part (c). This scoring gives
part (c) double weight relative to either part (a) or part (b).

Section 1 is essentially correct (E) if P( X > 3) is correctly computed and work is shown in part (a).

Section 1 is partially correct (P) if:


P( X ≥ 3) = 0.26 is computed;
OR
a correct numerical answer is given but no work is shown.

© 2007 The College Board. All rights reserved.


Visit apcentral.collegeboard.com (for AP professionals) and www.collegeboard.com/apstudents (for students and parents).
火箭学院 火速提分
新页码97

AP® STATISTICS
2007 SCORING GUIDELINES (Form B)

Question 2 (continued)

Section 2 is essentially correct (E) if in part (b):


1. the probability from part (a) is correctly used to calculate the probability that exactly 2 households are
in violation, either using the binomial pdf or using general probability rules, AND
2. work is shown.

Section 2 is partially correct (P) if in part (b):


the student computes P(Y ≥ 2 ) = 0.5270 or P(Y ≤ 2 ) = 0.7659 instead of P(Y = 2 ) ;
OR
the correct probability is given but no work is shown;
OR
the binomial coefficient is omitted ⎡⎣( 0.83) ( 0.17 ) = 0.0065⎤⎦ .
8 2

Section 3 is essentially correct (E) if the response to part (c):


recognizes that the distribution of X will be approximately normal;
OR
the response says that the distribution of X is more symmetric than the population distribution AND
mentions that the population distribution is highly skewed.

Section 3 is partially correct (P) if the response to part (c) reports a normal distribution for X without indicating
that the normal distribution is an approximation.

Section 4 is essentially correct (E) if the response to part (c) provides the appropriate mean μ X = μ = 1.65 and
σ 1.851
standard deviation σ X = = = 0.1511 for X .
n 150

Section 4 is partially correct (P) if the response to part (c):


provides either the correct mean or the correct standard deviation for X , but not both;
OR
provides correct numerical values for both the mean and standard deviation but sample notation
( X and s) is used instead of population notation (µ and σ X = σ / n );
OR
says only that the X distribution is centered in the same place as the population and has a smaller
standard deviation than the population (and does not give the values of 1.65 and 0.1511).

© 2007 The College Board. All rights reserved.


Visit apcentral.collegeboard.com (for AP professionals) and www.collegeboard.com/apstudents (for students and parents).
火箭学院 火速提分
新页码98

AP® STATISTICS
2007 SCORING GUIDELINES (Form B)

Question 2 (continued)

4 Complete Response

All four sections essentially correct

3 Substantial Response

Three sections essentially correct and no section partially correct


OR
Two sections essentially correct and two sections partially correct

2 Developing Response

Two sections essentially correct and no sections partially correct


OR
One section essentially correct and two sections partially correct
OR
No sections essentially correct and four sections partially correct

1 Minimal Response

One section essentially correct and no sections partially correct


OR
No sections essentially correct and two sections partially correct

If a response is between two scores (for example, 2½ points) use a holistic approach to determine whether
to score up or down depending on the strength of the response and communication. If the word
“approximately” is missing in part (c), round down.

© 2007 The College Board. All rights reserved.


Visit apcentral.collegeboard.com (for AP professionals) and www.collegeboard.com/apstudents (for students and parents).
火箭学院 火速提分
新页码99

AP® STATISTICS
2007 SCORING GUIDELINES (Form B)

Question 3

Intent of Question

The primary goals of this question are to: (1) assess a student’s ability to use blocking in designing an experiment,
and (2) describe a mechanism for randomly assigning treatments to experimental units in the context of the
selected blocking.

Solution

Part (a):

Acceptable blocking schemes:

Blocks: 1 and 12 2 and 3 4 and 5


6 and 7 8 and 9 10 and 11

Blocks: 1 and 12 2 and 5 3 and 4


6 and 7 8 and 11 9 and 10

Blocks: 1 and 12 2, 3, 4, and 5 6 and 7 8, 9, 10, and 11

We want to create blocks of homogeneous “units.” Exposure (side of house) would have an effect on heat
gain through a window, so the best blocking scheme would take side of house into account when creating
blocks.

The blocking schemes above create blocks that are similar with respect to exposure (side of house). Since
there are two treatments (types of windows), the optimal blocking scheme would create blocks consisting
of two window boxes each.

Part (b):

For each block we could select one of the window boxes and then flip a coin to determine which type of
window would be installed in that window box. For example, if the coin lands face up, install type A;
otherwise install type B. Continue this process until half of the windows in the block are assigned to one
type, then install the remaining window type in the other boxes.

Scoring

Each part is scored as either essentially correct (E), partially correct (P), or incorrect (I).

Part (a) is essentially correct (E) if:


1. one of the sets of blocks given in the solution is identified;
2. the justification for the blocking scheme demonstrates an understanding that windows of both types
should be used in equal numbers on each side of the house because of differing exposure to sun, light,
heat, etc.

Part (a) is partially correct (P) if it includes one of the two elements above.

© 2007 The College Board. All rights reserved.


Visit apcentral.collegeboard.com (for AP professionals) and www.collegeboard.com/apstudents (for students and parents).
火箭学院 火速提分
新页码100

AP® STATISTICS
2007 SCORING GUIDELINES (Form B)

Question 3 (continued)

Part (a) can be partially correct (P) if the student confuses treatments with blocks, for instance by assigning the
two “blocks” to be even- and odd-numbered window boxes. The justification must be that the exposure is similar
for the boxes on each side of the house, and thus each side must receive equal numbers of type A and type B
windows.

Part (b) is essentially correct (E) if it:


1. assigns window types at random to the window boxes in each block in a way that is consistent with
the blocks the student identifies in part (a) and that ensures an equal number of each type of window
within each block;
2. describes a mechanism for random assignment, such as a coin toss, roll of a die, use of random
number table, etc.

Part (b) is partially correct (P) if it includes only one of the two elements above.

NOTES:
• If students confuse treatments with blocks in part (a), they receive credit for the first element above only
if they are logically consistent. It is consistent if they use a scheme that randomly assigns half of each
type of window to go on each wall.

• By itself, “at random” is not sufficient as an answer and should be scored as incorrect.

• By itself, “at random within each block” is not sufficient for an essentially correct answer but can be
scored as partially correct.

4 Complete Response

Both parts essentially correct

3 Substantial Response

One part essentially correct and the other part partially correct

2 Developing Response

One part essentially correct and the other part incorrect


OR
Both parts partially correct

1 Minimal Response

One part partially correct

© 2007 The College Board. All rights reserved.


Visit apcentral.collegeboard.com (for AP professionals) and www.collegeboard.com/apstudents (for students and parents).
火箭学院 火速提分
新页码101

AP® STATISTICS
2007 SCORING GUIDELINES (Form B)

Question 4

Intent of Question

The goals of this question are to assess a student’s ability to: (1) plot a least squares regression line; (2) examine a
residual; and (3) discuss the effect of an additional observation on an estimated correlation coefficient and on the
least squares estimate of the slope of a line.

Solution

Parts (a) and (b):

When x = 67, ŷ = 35.1 + 0.427(67) = 63.709


and the residual = y - ŷ = 61 – 63.709 = –2.709.

Part (c):

See the new point indicated in the plot above. The slope would remain about the same since the new point
is consistent with the linear pattern in the original plot (i.e., close to the line).

sy
The correlation coefficient would increase. We know that b = r . The added point will increase sx
sx
sy
more than it will increase s y so will be less than 1. If the slope is to stay the same, r must increase.
sx
OR
This point fits the pattern well and has an x value that is far from x .

© 2007 The College Board. All rights reserved.


Visit apcentral.collegeboard.com (for AP professionals) and www.collegeboard.com/apstudents (for students and parents).
火箭学院 火速提分
新页码102

AP® STATISTICS
2007 SCORING GUIDELINES (Form B)

Question 4 (continued)

Scoring

This problem is scored in 4 sections. Section 1 consists of the graphical parts of (a) and (b) together. Section 2
consists of the numerical parts of (b). Section 3 consists of the first part of (c). Section 4 consists of the second
part of (c).

Each section is scored as either essentially correct (E), partially correct (P), or incorrect (I).

Section 1 (graphical parts of a and b) is essentially correct (E) if:


1. the regression line is drawn correctly on the scatterplot;
2. the point ( 67, 61) is circled and the vertical segment corresponding to the residual is drawn on the
scatterplot.

Section 1 is partially correct (P) if the response includes one of the above two elements.

Section 2 (numerical part of b) is essentially correct (E) if the residual is correctly computed as –2.709;
OR
the response states that the residual was approximated using the graph, a reasonable value for the residual is
given, and the sign of the residual is correct.

Section 2 is partially correct (P) if the magnitude of the residual is correct but the sign is wrong.

Section 3 (first part of (c)) is essentially correct (E) if it:


1. states that the slope will remain about the same (or change slightly);
2. provides an explanation based on the new point fitting the pattern in the original plot.

Section 3 is partially correct (P) if it states that the slope will be about the same, but the explanation is missing or
incorrect.

NOTE: If the line is drawn incorrectly in part (a), and the answer to this part is consistent with the line drawn,
section 3 is essentially correct (E).

Section 4 (second part of (c)) is essentially correct (E) if it:


1. states that the value of the correlation coefficient will increase;
2. provides an explanation based on the relative changes in sx and s y
OR
based on the fact that the new point fits the pattern AND is far out in the x direction,
OR
because the linear pattern is stronger.

Section 4 is partially correct (P) if it states that the value of the correlation coefficient will increase, but the
explanation is missing or incorrect.

NOTE: If the response just says that the correlation coefficient will increase because the point is close to the line,
section 4 is partially correct.

© 2007 The College Board. All rights reserved.


Visit apcentral.collegeboard.com (for AP professionals) and www.collegeboard.com/apstudents (for students and parents).
火箭学院 火速提分
新页码103

AP® STATISTICS
2007 SCORING GUIDELINES (Form B)

Question 4 (continued)

4 Complete Response

All four sections essentially correct

3 Substantial Response

Three sections essentially correct and no sections partially correct


OR
Two sections essentially correct and two sections partially correct

2 Developing Response

Two sections essentially correct and no sections partially correct


OR
One section essentially correct and two sections partially correct
OR
Four parts partially correct

1 Minimal Response

One section essentially correct and no sections partially correct


OR
No sections essentially correct and two sections partially correct

If a response is between two scores (for example, 2½ points), use a holistic approach to determine whether
to score up or down depending on the strength of the response and communication.

© 2007 The College Board. All rights reserved.


Visit apcentral.collegeboard.com (for AP professionals) and www.collegeboard.com/apstudents (for students and parents).
火箭学院 火速提分
新页码104

AP® STATISTICS
2007 SCORING GUIDELINES (Form B)

Question 5

Intent of Question

The primary intent of this question is to assess a student’s ability to make an inference about the difference in two
population means using a two-sample t-test, including the identification of the null and alternative hypotheses and
good communication of test results and conclusions.

Solution

Part 1: States a correct pair of hypotheses

H o : μ S − μ N = 0 versus H a : μ S − μ N < 0
OR
H o : μ N − μ S = 0 versus H a : μ N − μ S > 0
OR
H o : μ S = μ N versus H a : μ S < μ N

where ms = mean decrease in cholesterol for standard drug


and mN = mean decrease in cholesterol for new drug.

Part 2: Identifies a correct test (by name or by formula) and checks appropriate assumptions.

XS − XN
Two-sample t-test (or z-test) t=
sS2 sN2
+
nS nN

Assumptions:
1. random assignment of subjects to treatments;
2. normal population distributions or large samples.

Both sample sizes were large (50), and there was random assignment of subjects to treatments.

NOTE: A two-sample z-test is acceptable as long as the large sample sizes are noted. A pooled t-test is also
acceptable, but the student must also state and comment on the plausibility of the equal population variances
assumption.

Part 3: Correct mechanics, including the value of the test statistic, df (stated or implied by calculator work), and
p-value (or rejection region).

XS − XN 10 − 18
t= = = −3.92
2 2
s s 82 122
S
+ N
+
nS nN 50 50

© 2007 The College Board. All rights reserved.


Visit apcentral.collegeboard.com (for AP professionals) and www.collegeboard.com/apstudents (for students and parents).
火箭学院 火速提分
新页码105

AP® STATISTICS
2007 SCORING GUIDELINES (Form B)
Question 5 (continued)

df = 85, p-value = 0.000088, or from table p-value < 0.001


OR
df = 49, p-value = 0.00014.
For two-sample z-test, z = -3.92, , p-value = 0.000044.
For pooled t-test, t = -3.92, df = 98, p-value = 0.000081.

Rejection regions:

a = 0.10 : t < -1.303 (df = 40), t < -1.292 (df = 80), t < -1.290 (df = 100) OR z < -1.28
a = 0.05 : t < -1.684 (df = 40), t < -1.664 (df = 80), t < -1.660 (df = 100) OR z < -1.645
a = 0.01 : t < -2.423 (df = 40), t < -2.374 (df = 80), t < -2.364 (df = 100) OR z < -2.33

Part 4: Stating a correct conclusion in the context of the problem, using the result of the statistical test.

Because the p-value < selected α (or because the p-value is so small), reject Ho. There is convincing
evidence that the mean cholesterol reduction is greater for the new drug.

Scoring

Each part is scored as either essentially correct (E), partially correct (P), or incorrect (I).

Part 1 is essentially correct (E) if the response:


1. includes the correct pair of hypotheses;
2. defines the parameters in the hypotheses in the context of the problem.

Part 1 is partially correct (P) if the hypotheses are stated correctly, but notation is not defined.

Part 2 is essentially correct (E) if the response:


1. identifies the correct test by name or formula;
2. checks appropriate assumptions (including equal variance if pooled t-test is used).

Part 2 is partially correct (P) if it includes only one of the two elements above.

Part 3 is essentially correct (E) if the response includes:


1. correct mechanics, including the value of the test statistic;
2. df and p-value or rejection region consistent with the hypotheses in part 1.

Part 3 is partially correct (P) if it includes only one of the two elements above.

Part 4 is essentially correct (E) if the response includes:


1. a conclusion in context consistent with the hypotheses in part 1;
2. linkage between the results of the test in part 3 and the conclusion, and this is communicated well.

Part 4 is partially correct (P) if it includes only one of the two elements above.

© 2007 The College Board. All rights reserved.


Visit apcentral.collegeboard.com (for AP professionals) and www.collegeboard.com/apstudents (for students and parents).
火箭学院 火速提分
新页码106

AP® STATISTICS
2007 SCORING GUIDELINES (Form B)

Question 5 (continued)

NOTES:
• If both an α and a p-value are given, the linkage is implied. If no α is given, the solution must be explicit
about the linkage by giving a correct interpretation of the p-value or explaining how the conclusion
follows from the p-value.
• If the p-value in part 3 is incorrect but the conclusion is consistent with the computed p-value, part 4 can
be considered essentially correct (E).

4 Complete Response

All four parts essentially correct

3 Substantial Response

Three parts essentially correct and no parts partially correct


OR
Two parts essentially correct and two parts partially correct

2 Developing Response

Two parts essentially correct and no parts partially correct


OR
One part essentially correct and two parts partially correct
OR
Four parts partially correct

1 Minimal Response

One part essentially correct and no parts partially correct


OR
No parts essentially correct and two parts partially correct

If a response is between two scores (for example, 2½ points), use a holistic approach to determine whether
to score up or down depending on the strength of the response and communication.

© 2007 The College Board. All rights reserved.


Visit apcentral.collegeboard.com (for AP professionals) and www.collegeboard.com/apstudents (for students and parents).
火箭学院 火速提分
新页码107

AP® STATISTICS
2007 SCORING GUIDELINES (Form B)

Question 6

Intent of Question

The primary intent of this question is to assess a student’s ability to: (1) make an inference about the difference in
two population proportions; (2) examine a regression model for a linear trend in proportions; and (3) construct a
confidence interval for a slope. The investigative part of this question requires a student to use a regression model
to estimate survival probabilities for two different situations and make an inference about the expected number of
surviving species that would be achieved.

Solution

Part (a):

Part 1: States a correct pair of hypotheses

H o : pL − pS = 0 versus H a : pL − pS < 0
OR
H o : pS − pL = 0 versus H a : pS − pL > 0
OR
H o : pL = pS versus H a : pL < pS

Where

pL is the proportion going extinct on large islands,


and
pS is the proportion going extinct on small islands.

Part 2: Identifies a correct test (by name or by formula) and checks appropriate assumptions.

Two-sample test for proportions

pˆ L − pˆ S
z=
pˆ (1 − pˆ ) pˆ (1 − pˆ )
+
nL nS

Assumptions: independent observations and large sample sizes.

The problem states that whether one species becomes extinct is independent of whether another species
becomes extinct, and that the probability of extinction is the same for all species on large islands and for
all species on small islands, so it is reasonable to assume that observations are independent.

© 2007 The College Board. All rights reserved.


Visit apcentral.collegeboard.com (for AP professionals) and www.collegeboard.com/apstudents (for students and parents).
火箭学院 火速提分
新页码108

AP® STATISTICS
2007 SCORING GUIDELINES (Form B)

Question 6 (continued)

pˆ L = 0.091 pˆ S = 0.221
nL pˆ L = 19 nL (1 − pˆ L ) = 189
nS pˆ S = 66 nS (1 − pˆ S ) = 233

All are greater than 5 (or 10), so the sample sizes are large enough to proceed.

Part 3: Correct mechanics, including the value of the test statistic and p-value (or rejection region).

19 + 66 85
pˆ = = = 0.168
208 + 299 507

pˆ L − pˆ S 0.091 − 0.221 −0.130


z= = = = −3.82
pˆ (1 − pˆ ) pˆ (1 − pˆ ) (0.168)(0.832) (0.168)(0.832) 0.034
+ +
nL nS 208 299

p-value = 0.00006

(from table p-value ≈ 0; graphing calculator: z = -3.836233478 , p-value = 0.00006)

Part 4: Stating a correct conclusion in the context of the problem, using the result of the statistical test.

Because the p-value is less than the stated α (or because the p-value is so small, or because the test
statistic is in the rejection region), reject H o . There is sufficient evidence that the proportion of species
becoming extinct is smaller for large islands than for small islands.

If both an a and a p-value are given, the linkage is implied. If no a is given, the solution must be
explicit about the linkage by giving a correct interpretation of the p-value or explaining how the
conclusion follows from the p-value.

If the p-value in part 3 is incorrect but the conclusion is consistent with the computed p-value, part 4 can
be considered as correct.

Part (b):

Compute a 95 percent confidence interval for the slope of the regression line.

Part 1: Identifies appropriate confidence interval by name or by formula.

The confidence interval for the slope of the regression line is b ± tsb .

Part 2: Checks appropriate assumptions.

Assumptions: The residual plot shows no unusual patterns that would suggest violation of the
assumptions, so it is reasonable to proceed.

© 2007 The College Board. All rights reserved.


Visit apcentral.collegeboard.com (for AP professionals) and www.collegeboard.com/apstudents (for students and parents).
火箭学院 火速提分
新页码109

AP® STATISTICS
2007 SCORING GUIDELINES (Form B)
Question 6 (continued)

Part 3: Correct mechanics.

df = n – 2 = 13-2 = 11
−0.05323 ± 2.20(0.00618)
−0.05323 ± 0.013596
(−0.0668, −0.0396)

Part 4: Interpretation.

We are 95 percent confident that the mean proportion of species going extinct decreases by somewhere
between 0.03 and 0.06 with each increase of 1 unit in ln(area). The proportion of species going extinct
decreases with increasing area.

Part (c):

From part (b) it appears that the proportion of species going extinct decreases with increasing area.
Therefore the proportion of species going extinct is related to the size of the island. Because the island
sizes differed within the large island group and within the small island group, the assumption is probably
not reasonable.

Scoring

Each part is scored as either essentially correct (E), partially correct (P), or incorrect (I).

Part (a) is essentially correct (E) if three or four parts of the hypothesis test are correct.

Part (a) is partially correct (P) if one or two parts of the hypothesis test are correct.

NOTE: For part 2 of (a), the independent observations assumption does not have to be addressed in the response
to get credit for this part, since this is given in the stem of the problem.

Part (b) is essentially correct (E) if three or four parts of the confidence interval are correct.

Part (b) is partially correct (P) if one or two parts of the confidence interval are correct.

Part (c) is essentially correct (E) if the response:


1. states the assumptions are not reasonable, AND
2. gives a justification based on the information in part (b);
OR
says that the assumptions are reasonable based on an incorrect conclusion in part (b) that island size
is not related to extinction proportion, with an appropriate explanation.

© 2007 The College Board. All rights reserved.


Visit apcentral.collegeboard.com (for AP professionals) and www.collegeboard.com/apstudents (for students and parents).
火箭学院 火速提分
新页码110

AP® STATISTICS
2007 SCORING GUIDELINES (Form B)

Question 6 (continued)

Part (c) is partially correct (P) if:


it says that the assumption is not reasonable, but the explanation is weak or does not appeal to the
information in part (b);
OR
it says that the assumption is not reasonable because the negative estimate of the slope given in part (b) is
misinterpreted to suggest that survival rates decrease as area increases;
OR
it appeals to part (b) but says that the assumption is reasonable because within each group (large/small),
the island sizes don’t vary too much;
OR
it says the assumption is reasonable because the negative estimate of the slope given in part (b) is
misinterpreted;
OR
the justification appeals to the differing proportions in the original data table only.

Part (c) is incorrect if a choice is made but no justification is given.

Part (d) is essentially correct (E) if the large preserve is chosen and the decision is well supported based on the
expectation that a larger number of species will be preserved, in comparison to the expected number preserved on
the five small islands.

Part (d) is partially correct (P) if:


the large preserve is chosen based on the results from parts (a) and/or (b);
OR
the large preserve is chosen but the justification is weak;
OR
the five small preserves are chosen based on an incorrect computation of the number of species saved for
the two scenarios.

Part (d) is incorrect if:


a choice is made (large or five small) but no justification is given;
OR
five small preserves are chosen based only on the fact that there are 80 rather than 70 species at the
outset.

© 2007 The College Board. All rights reserved.


Visit apcentral.collegeboard.com (for AP professionals) and www.collegeboard.com/apstudents (for students and parents).
火箭学院 火速提分
新页码111

AP® STATISTICS
2007 SCORING GUIDELINES (Form B)

Question 6 (continued)

4 Complete Response

All four parts essentially correct

3 Substantial Response

Three parts essentially correct and no parts partially correct


OR
Two parts essentially correct and two parts partially correct

2 Developing Response

Two parts essentially correct and no parts partially correct


OR
One part essentially correct and two parts partially correct
OR
Four parts partially correct

1 Minimal Response

One part essentially correct and no parts partially correct


OR
No parts essentially correct and two parts partially correct

If a response is between two scores (for example, 2½ points), use a holistic approach to determine whether
to score up or down depending on the strength of the response and communication.

© 2007 The College Board. All rights reserved.


Visit apcentral.collegeboard.com (for AP professionals) and www.collegeboard.com/apstudents (for students and parents).
关注“火箭学院”公众号

获取第一手
“课程热点资讯”

扫描二维码添加“火箭队长”

即刻获得
“资料真题大礼包”

You might also like